Параллельное подключение резисторов формула. Параллельное соединение резисторов: формулы, расчеты и примеры

Как рассчитать параллельное соединение резисторов. Какие формулы использовать для вычисления общего сопротивления. Чем отличается от последовательного соединения. Как подобрать нужный номинал резистора с помощью параллельного соединения.

Содержание

Что такое параллельное соединение резисторов

Параллельное соединение резисторов — это способ подключения нескольких резисторов, при котором их выводы соединяются между собой следующим образом:

  • Все левые выводы резисторов соединяются в одной точке
  • Все правые выводы резисторов соединяются в другой точке

Схематически параллельное соединение резисторов выглядит следующим образом:

«`
R1 R2 R3 «`

Какие особенности имеет параллельное соединение резисторов.

Напряжение на резисторах

При параллельном соединении напряжение на всех резисторах одинаково и равно напряжению источника питания:

U = U1 = U2 = U3 = … = Un

Где:

  • U — напряжение источника питания
  • U1, U2, U3, …, Un — напряжения на отдельных резисторах

Ток через резисторы

Общий ток в цепи равен сумме токов через отдельные резисторы:


I = I1 + I2 + I3 + … + In

Где:

  • I — общий ток в цепи
  • I1, I2, I3, …, In — токи через отдельные резисторы

Формула расчета общего сопротивления при параллельном соединении

Для расчета общего сопротивления при параллельном соединении резисторов используется следующая формула:

1/R = 1/R1 + 1/R2 + 1/R3 + … + 1/Rn

Где:

  • R — общее сопротивление
  • R1, R2, R3, …, Rn — сопротивления отдельных резисторов

Из этой формулы можно вывести выражение для общего сопротивления:

R = 1 / (1/R1 + 1/R2 + 1/R3 + … + 1/Rn)

Формула для двух параллельных резисторов

Для случая двух параллельно соединенных резисторов формула упрощается:

R = (R1 * R2) / (R1 + R2)

Примеры расчета параллельного соединения резисторов

Пример 1: Два параллельных резистора

Допустим, у нас есть два резистора с сопротивлениями 100 Ом и 200 Ом. Рассчитаем их общее сопротивление при параллельном соединении:

R = (100 * 200) / (100 + 200) = 20000 / 300 ≈ 66.7 Ом

Пример 2: Три параллельных резистора

Рассмотрим три параллельно соединенных резистора: 10 Ом, 20 Ом и 30 Ом. Рассчитаем общее сопротивление:


1/R = 1/10 + 1/20 + 1/30 = 0.1 + 0.05 + 0.033 = 0.183

R = 1 / 0.183 ≈ 5.46 Ом

Чем отличается параллельное соединение от последовательного

Основные отличия параллельного соединения резисторов от последовательного:

  1. При параллельном соединении напряжение на всех резисторах одинаково, а при последовательном — разное.
  2. Общий ток при параллельном соединении равен сумме токов через резисторы, а при последовательном — одинаков для всех резисторов.
  3. Общее сопротивление при параллельном соединении всегда меньше сопротивления самого маленького резистора, а при последовательном — больше суммы сопротивлений всех резисторов.

Как подобрать нужный номинал с помощью параллельного соединения

Параллельное соединение резисторов часто используется для получения нестандартного номинала сопротивления. Рассмотрим пример:

Допустим, нам нужен резистор на 80 Ом, но в наличии есть только резисторы на 100 Ом и 400 Ом. Можно ли получить нужное сопротивление?

Воспользуемся формулой для двух параллельных резисторов:


R = (R1 * R2) / (R1 + R2) = (100 * 400) / (100 + 400) = 40000 / 500 = 80 Ом

Таким образом, соединив параллельно резисторы 100 Ом и 400 Ом, мы получим требуемое сопротивление 80 Ом.

Практическое применение параллельного соединения резисторов

Параллельное соединение резисторов широко применяется в электронике и электротехнике. Вот несколько примеров:

  • Деление токов в электрических цепях
  • Создание резистивных делителей напряжения
  • Получение нестандартных номиналов сопротивлений
  • Увеличение мощности рассеивания в цепи
  • Создание эквивалентных сопротивлений в сложных схемах

Особенности параллельного соединения резисторов

Преимущества параллельного соединения

Параллельное соединение резисторов имеет ряд преимуществ:

  • Возможность получения меньшего общего сопротивления, чем у отдельных резисторов
  • Увеличение общей мощности рассеивания
  • Продолжение работы цепи при выходе из строя одного из резисторов
  • Гибкость в подборе нужного сопротивления

Недостатки параллельного соединения

У параллельного соединения есть и некоторые недостатки:


  • Сложность расчетов при большом количестве резисторов
  • Увеличение общего тока в цепи
  • Необходимость использования резисторов с большим номинальным сопротивлением для получения малых сопротивлений

Заключение

Параллельное соединение резисторов — важный инструмент в арсенале электронщика и радиолюбителя. Понимание принципов работы такого соединения и умение производить расчеты позволяет решать широкий спектр задач при проектировании и ремонте электронных устройств.

Основные моменты, которые следует помнить о параллельном соединении резисторов:

  • Напряжение на всех резисторах одинаково
  • Общий ток равен сумме токов через отдельные резисторы
  • Общее сопротивление всегда меньше сопротивления самого маленького резистора
  • Для расчета общего сопротивления используется формула 1/R = 1/R1 + 1/R2 + 1/R3 + … + 1/Rn

Освоив принципы параллельного соединения резисторов, вы сможете более эффективно проектировать электронные схемы и решать задачи, связанные с расчетом сопротивлений в сложных цепях.



формула и примеры расчета сопротивления, напряжения, тока и мощности

При проектировании электрических схем возникает необходимость использования последовательного и параллельного соединений резисторов. Соединения применяются также и при ремонтах электрооборудования, поскольку в некоторых ситуациях невозможно найти эквивалентный номинал резистора. Выполнить расчет просто, и справиться с этой операцией может каждый.

Типы проводников

Проводимость веществом электрического тока связана с наличием в нем свободных носителей заряда. Их количество определяется по электронной конфигурации. Для этого необходима химическая формула вещества, при помощи которой можно вычислить их общее число. Значение для каждого элемента берется из периодической системы Дмитрия Ивановича Менделеева.

Электрический ток — упорядоченное движение свободных носителей заряда, на которые воздействует электромагнитное поле. При протекании тока по веществу происходит взаимодействие потока заряженных частиц с узлами кристаллической решетки, при этом часть кинетической энергии частицы превращается в тепловую энергию. Иными словами, частица «ударяется» об атом, а затем снова продолжает движение, набирая скорость под действием электромагнитного поля.

Процесс взаимодействия частиц с узлами кристаллической решетки называется электрической проводимостью или сопротивлением материала. Единицей измерения является Ом, а определить его можно при помощи омметра или расчитать. Согласно свойству проводимости, вещества можно разделить на 3 группы:

  1. Проводники (все металлы, ионизированный газ и электролитические растворы).
  2. Полупроводники (Si, Ge, GaAs, InP и InSb).
  3. Непроводники (диэлектрики или изоляторы).

Проводники всегда проводят электрический ток, поскольку содержат в своем атомарном строении свободные электроны, анионы, катионы и ионы. Полупроводники проводят электричество только при определенных условиях, которые влияют на наличие или отсутствие свободных электронов и дырок. К факторам, влияющим на проводимость, относятся следующие: температура, освещенность и т. д. Диэлектрики вообще не проводят электричество, поскольку в их структуре вообще отсутствуют свободные носители заряда. При выполнении расчетов каждый радиолюбитель должен знать зависимость сопротивления от некоторых физических величин.

Зависимость сопротивления

Значение электропроводимости зависит от нескольких факторов, которые необходимо учитывать при расчетах, изготовлении элементов резистивной нагрузки (резисторов), ремонте и проектировании устройств. К этим факторам необходимо отнести следующие:

  1. Температура окружающей среды и материала.
  2. Электрические величины.
  3. Геометрические свойства вещества.
  4. Тип материала, из которого изготовлен проводник (полупроводник).

К электрическим величинам можно отнести разность потенциалов (напряжение), электродвижущую силу (ЭДС) и силу тока. Геометрией проводника является его длина и площадь поперечного сечения.

Электрические величины

Зависимость величины электропроводимости от параметров электричества определяется законом Ома. Существует две формулировки: одна — для участка, а другая — для полной цепи. В первом случае соотношение определяются, исходя из значений силы тока (I) и напряжения (U) простой формулой: I = U / R. Из соотношения видна прямо пропорциональная зависимость тока от величины напряжения, а также обратно пропорциональная от сопротивления. Можно выразить R: R = U / I.

Для расчета электропроводимости всего участка следует воспользоваться соотношением между ЭДС (e), силой тока (i), а также внутренним сопротивлением источника питания (Rвн): i = e / (R+Rвн). В этом случае величина R вычисляется по формуле: R = (e / i) — Rвн. Однако при выполнении расчетов необходимо учитывать также геометрические параметры и тип проводника, поскольку они могут существенно повлиять на вычисления.

Тип и геометрические параметры

Свойство вещества к проводимости электричества определяется структурой кристаллической решетки, а также количеством свободных носителей. Исходя из этого, тип вещества является ключевым фактором, который определяет величину электропроводимости. В науке коэффициент, определяющий тип вещества, обозначается литерой «р» и называется удельным сопротивлением. Его значение для различных материалов (при температуре +20 градусов по Цельсию) можно найти в специальных таблицах.

Иногда для удобства расчетов используется обратная величина, которая называется удельной проводимостью (σ). Она связана с удельным сопротивлением следующим соотношением: p = 1 / σ. Площадь поперечного сечения (S) влияет на электрическое сопротивление. С физической точки зрения, зависимость можно понять следующим образом: при малом сечении происходят более частые взаимодействия частиц электрического тока с узлами кристаллической решетки. Поперечное сечение можно вычислить по специальному алгоритму:

  1. Измерение геометрических параметров проводника (диаметр или длину сторон) при помощи штангенциркуля.
  2. Визуально определить форму материала.
  3. Вычислить площадь поперечного сечения по формуле, найденной в справочнике или интернете.

В случае когда проводник имеет сложную структуру, необходимо вычислить величину S одного элемента, а затем умножить результат на количество элементов, входящих в его состав. Например, если провод является многожильным, то следует вычислить S для одной жилы. После этого нужно умножить, полученную величину S, на количество жил. Зависимость R от вышеперечисленных величин можно записать в виде соотношения: R = p * L / S. Литера «L» является длиной проводника. Однако для получения точных расчетов необходимо учитывать температурные показатели внешней среды и проводника.

Температурные показатели

Существует доказательство зависимости удельного сопротивления материала от температуры, основанное на физическом эксперименте. Для проведения опыта нужно собрать электрическую цепь, состоящую из следующих элементов: источника питания, нихромовой спирали, соединительных проводов амперметра и вольтметра. Приборы нужны для измерения значений силы тока и напряжения соответственно. При протекании электричества происходит нагревание нихромовой пружины. По мере ее нагревания, показания амперметра уменьшаются. При этом происходит существенное падение напряжения на участке цепи, о котором свидетельствуют показания вольтметра.

В радиотехнике уменьшение величины напряжение называется просадкой или падением. Формула зависимости р от температуры имеет следующий вид: p = p0 * [1 + a * (t — 20)]. Значение p0 — удельное сопротивление материала, взятого из таблицы, а литера «t» — температура проводника.

Температурный коэффициент «а» принимает следующие значения: для металлов — a>0, а для электролитических растворов — a<0. Для получения формулы, определяющей все зависимости, необходимо подставить все соотношения в общую формулу зависимости R от типа материала, температуры, длины и сечения: R = p0 * [1 + a * (t — 20)] * L / S. Формулы используются только для расчетов и изготовления резисторов. Для быстрого измерения величины сопротивления применяется омметр.

Объединение резистивных радиокомпонентов

Для получения необходимого номинала сопротивления применяются два типа соединения резисторов: параллельное и последовательное. Если их соединить параллельно, то нужно два вывода одного резистора подключить к двум выводам другого. Если соединение является последовательным, то один вывод резистора соединяется с одним выводом другого резистора. Соединения используются для получения необходимых номиналов сопротивлений, а также для увеличения рассеивания мощности тока, протекающего по цепи.

Каждое из соединений обладает определенными характеристиками. Кроме того, последовательно или параллельно могут объединяться несколько резисторов. Соединения также могут быть смешанными, т. е. применяться оба типа объединения радиокомпонентов.

Параллельное соединение

При параллельном подключении значение напряжения на всех резисторах одинаковое, а сила тока — обратно пропорциональна их общему сопротивлению. В интернете web-разработчики создали для расчета величины общего сопротивления параллельного соединения резисторов онлайн-калькулятор.

Рассчитывается общее сопротивление при параллельном соединении по формуле: 1 / Rобщ = (1 / R1) + (1 / R2) + …+ (1 / Rn). Если выполнить математические преобразования и привести к общему знаменателю, то получится удобная формула параллельного соединения для расчета Rобщ. Она имеет следующий вид: Rобщ = (R1 * R2 * … * Rn) / (R1 + R2 + … + Rn). Если необходимо рассчитать величину Rобщ только для двух радиокомпонентов, то формула параллельного сопротивления имеет следующий вид: Rобщ = (R1 * R2) / (R1 + R2).

При ремонте или проектировании схемы устройства возникает задача объединения нескольких резистивных элементов для получения конкретной величины сопротивления. Например, значение Rобщ для определенной цепочки элементов равно 8 Ом, которое получено при расчетах. Перед радиолюбителем стоит задача, какие нужно подобрать номиналы для получения нужного значения (в стандартном ряду резисторов отсутствует радиокомпонент с номиналом в 8 Ом, а только 7,5 и 8,2). В этом случае нужно найти сопротивление при параллельном соединении резистивных элементов. Посчитать значение Rобщ для двух элементов можно следующим образом:

  1. Номинал резистора в 16 Ом подойдет.
  2. Подставить в формулу: R = (16 * 16) / (16 + 16) = 256 / 32 = 8 (Ом).

В некоторых случаях следует потратить больше времени на подбор необходимых номиналов. Можно применять не только два, но и три элемента. Сила тока вычисляется с использованием первого закона Кирхгофа. Формулировка закона следующая: общее значение тока, входящего и протекающего по цепи, равен выходному его значению. Величина силы тока для цепи, состоящей из двух резисторов (параллельное соединение) рассчитывается по такому алгоритму:

  1. Ток, протекающий через R1 и R2: I1 = U / R1 и I2 = U / R2 соответственно.
  2. Общий ток — сложение токов на резисторах: Iобщ = I1 + I2.

Например, если цепь состоит из 2 резисторов, соединенных параллельно, с номиналами в 16 и 7,5 Ом. Они запитаны от источника питания напряжением в 12 В. Значение силы тока на первом резисторе вычисляется следующим способом: I1 = 12 / 16 = 0,75 (А). На втором резисторе ток будет равен: I2 = 12 / 7,5 = 1,6 (А). Общий ток определяется по закону Кирхгофа: I = I1 + I2 = 1,6 + 0,75 = 2,35 (А).

Последовательное подключение

Последовательное включение резисторов также применяется в радиотехнике. Методы нахождения общего сопротивления, напряжения и тока отличаются от параллельного подключения. Основные правила соединения следующие:

  1. Ток не изменяется на участке цепи.
  2. Общее напряжение равно сумме падений напряжений на каждом резисторе.
  3. Rобщ = R1 + R2 + … + Rn.

Пример задачи следующий: цепочка, состоящая из 2 резисторов (16 и 7,5 Ом), питается от источника напряжением 12 В и током в 0,5 А. Необходимо рассчитать электрические параметры для каждого элемента. Порядок расчета следующий:

  1. I = I1 = I2 = 0,5 (А).
  2. Rобщ = R1 + R2 = 16 + 7,5 = 23,5 (Ом).
  3. Падения напряжения: U1 = I * R1 = 0,5 * 16 = 8 (В) и U2 = I * R2 = 0,5 * 7,5 = 3,75 (В).

Не всегда выполняется равенство напряжений (12 В не равно 8 + 3,75 = 11,75 В), поскольку при этом расчете не учитывается сопротивление соединительных проводов. Если схема является сложной, и в ней встречается два типа соединений, то нужно выполнять расчеты по участкам. В первую очередь, рассчитать для параллельного соединения, а затем для последовательного.

Таким образом, параллельное и последовательное соединения резисторов применяются для получения более точных значений сопротивлений, а также при отсутствии необходимого номинала радиокомпонента при проектировании или ремонте устройств.

Параллельное соединение резисторов. Калькулятор для расчета

Параллельное соединение резисторов — одно из двух видов электрических соединений, когда оба вывода одного резистора соединены с соответствующими выводами другого резистора или резисторов. Зачастую резисторы соединяют последовательно или параллельно для того, чтобы создать более сложные электронные схемы.

Схема параллельного соединения резисторов показан на рисунке ниже. При параллельном соединении резисторов, напряжение на всех резисторах будет одинаковым, а протекающий через них ток будет пропорционален их сопротивлению:

Формула параллельного соединения резисторов

Общее сопротивление нескольких резисторов соединенных параллельно определяется по следующей формуле:

Ток, протекающий через отдельно взятый резистор, согласно закону Ома, можно найти по формуле:

Параллельное соединение резисторов — расчет

Пример  №1

При разработке устройства, возникла необходимость установить резистор с сопротивлением 8 Ом. Если мы просмотрим весь номинальный ряд стандартных значений резисторов, то мы увидим, что резистора с сопротивлением в 8 Ом в нем нет.

Выходом из данной ситуации будет использование двух параллельно соединенных резисторов. Эквивалентное значение сопротивления для двух резисторов соединенных параллельно рассчитывается следующим образом:

Профессиональный цифровой осциллограф

Количество каналов: 1, размер экрана: 2,4 дюйма, разрешен…

Данное уравнение показывает, что если R1 равен R2, то сопротивление R составляет половину сопротивления одного из двух резисторов. При R = 8 Ом, R1 и R2 должны, следовательно, иметь значение 2 × 8 = 16 Ом.
Теперь проведем проверку, рассчитав общее сопротивление двух резисторов:

Таким образом, мы получили необходимое сопротивление 8 Ом, соединив параллельно два резистора по 16 Ом.

Пример расчета №2

Найти общее сопротивление  R из трех параллельно соединенных резисторов:

Общее сопротивление R рассчитывается по формуле:

Этот метод расчета может быть использованы для расчета любого количества отдельных сопротивлений соединенных параллельно.

Один важный момент, который необходимо запомнить при расчете параллельно соединенных резисторов – это то, что общее сопротивление всегда будет меньше, чем значение наименьшего сопротивления в этой комбинации.

Как рассчитать сложные схемы соединения резисторов

Более сложные соединения резисторов могут быть рассчитаны путем систематической группировки резисторов. На рисунке ниже необходимо посчитать общее сопротивление цепи, состоящей из трех резисторов:


Для простоты расчета, сначала сгруппируем резисторы по параллельному и последовательному типу соединения.
Резисторы R2 и R3 соединены последовательно (группа 2). Они в свою очередь соединены параллельно с резистором R1 (группа 1).

Последовательное соединение резисторов группы 2 вычисляется как сумма сопротивлений R2 и R3:

В результате мы упрощаем схему в виде двух параллельных резисторов. Теперь общее сопротивление всей схемы можно посчитать следующим образом:

Расчет более сложных соединений резисторов можно выполнить используя законы Кирхгофа.

Ток, протекающий в цепи параллельно соединенных резисторах

Общий ток I протекающий в цепи параллельных резисторов равняется сумме отдельных токов, протекающих во всех параллельных ветвях, причем ток в отдельно взятой ветви не обязательно должен быть равен току в соседних ветвях.

Несмотря на параллельное соединение, к каждому резистору приложено одно и то же напряжение. А поскольку величина сопротивлений в параллельной цепи может быть разной, то и величина протекающего тока через каждый резистор тоже будет отличаться (по определению закона Ома).

Рассмотрим это на примере двух параллельно соединенных резисторов. Ток, который течет через каждый из резисторов ( I1 и I2 ) будет отличаться друг от друга поскольку сопротивления резисторов R1 и R2 не равны.
Однако мы знаем, что ток, который поступает в цепь в точке «А» должен выйти из цепи в точке «B» .

Правило Кирхгофа гласит: «Общий ток, входящий в цепь равен току выходящему из цепи».

Таким образом, протекающий общий ток в цепи  можно определить как:

I = I1 + I2

Затем с помощью закона Ома можно вычислить ток, который протекает через каждый резистор:

Ток, протекающий в R1 = U ÷ R1 = 12 ÷ 22 кОм = 0,545 мА

Ток, протекающий в R 2 = U ÷ R2 = 12 ÷ 47 кОм = 0,255 мА

Таким образом, общий ток будет равен:

I = 0,545 мА + 0,255 мА = 0,8 мА

Это также можно проверить, используя закон Ома:

I = U ÷ R = 12 В ÷ 15 кОм = 0,8 мА (то же самое)

где 15кОм — это общее сопротивление двух параллельно соединенных резисторов (22 кОм и 47 кОм)

И в завершении хочется отметить, что большинство современных резисторов маркируются цветными полосками и назначение ее можно узнать здесь.

Параллельное соединение резисторов — онлайн калькулятор

Чтобы быстро вычислить общее сопротивление двух и более резисторов, соединенных параллельно, вы можете воспользоваться следующим онлайн калькулятором:

Подведем итог

Когда два или более резистора соединены так, что оба вывода одного резистора соединены с соответствующими выводами другого резистора или резисторов, то говорят, что они соединены между собой параллельно. Напряжение на каждом резисторе внутри параллельной комбинации одинаковое, но токи, протекающие через них, могут отличаться друг от друга, в зависимости от величины сопротивлений каждого резистора.

Эквивалентное или полное сопротивление параллельной комбинации всегда будет меньше минимального сопротивления резистора, входящего в параллельное соединение.

Параллельное соединение резисторов: расчет и формулы

В случае последовательного соединения прохождение тока осуществляется только через один проводник. Параллельное соединение резисторов предполагает распределение электрического тока среди нескольких проводников. При добавлении еще одного резистора в электрическую цепь, ток будет частично проходить через разные резисторы.

Схемы последовательного и параллельного соединения

Если рассматривать соединение на примере громкоговорителя, то при последовательном соединении с усилителем мощности подключается только один динамик, поскольку прохождение тока осуществляется только через один проводник. Подключение второго громкоговорителя может быть выполнено разными способами.

При последовательном соединении по обоим устройствам будет протекать одинаковый ток. В этом случае общее сопротивление приборов представляет собой сумму отдельно взятых сопротивлений.

При параллельном соединении протекание тока будет происходить по двум направлениям. Здесь общее значение сопротивления в отличие от последовательного соединения, наоборот, будет уменьшаться. То есть, при параллельном соединении двух сопротивлений, их общее значение будет составлять половину каждого из них.

Если последовательное и параллельное соединение резисторов рассматривается с точки зрения радиоэлектроники, необходимо четко представлять себе, что представляет собой данный элемент и какова его роль в электронных схемах. Эта деталь является неотъемлемой частью многих устройств, благодаря такому свойству, как сопротивление электрическому току. Резисторы могут быть двух типов – постоянными и переменными, то есть подстроечными. При создании тех или иных электрических схем требуется резистор установленного номинала, которого в данный момент может не оказаться в наличии. Поэтому приходится использовать элементы с другими номинальными значениями, формула для каждого из которых подтверждает их физические свойства.

Последовательное соединение считается наиболее простым. Оно используется, когда необходимо увеличить общее сопротивление электрической цепи. В этом случае все сопротивления резисторов просто складываются и дают общую сумму. При параллельном соединении, наоборот, можно снизить результирующее сопротивление или увеличить мощность за счет нескольких подключенных резисторов.

Отличие параллельного и последовательного соединения

Последовательное и параллельное соединение резисторов отличаются между собой значениями напряжения. В каждой части параллельных контуров этот показатель будет одинаковым. Однако, при одном и том же напряжении, сила тока в контурах будет разной. Кроме того, сопротивление резисторов при параллельном соединении будет существенно отличаться от того же показателя при последовательном соединении.

В процессе использования последовательной схемы наблюдаются обратные явления. Сила тока в каждом сопротивлении будет одна и та же, а напряжение на каждом участке будет отличаться. Это связано с тем, что во время протекания тока, каждый резистор частично забирает приложенное напряжение. Из-за различного сопротивления резисторов, при последовательном соединении, напряжение в цепи может падать. Для того чтобы подтвердить данное явление, выполняется расчет сопротивления. Все падения напряжения в общей сумме равняются общему напряжению, которое было приложено. Для проведения вычислений используются формулы, с помощью которых можно получить наиболее точные результаты.

Таким образом, параллельное соединение резисторов, находящихся под одинаковым напряжением, не влияет на режим работы каждого из них. То есть, они совершенно не зависят друг от друга, и ток, проходящий по одному приемнику, не может существенно влиять на другие приемники.

Формула расчета параллельного соединения резисторов

Свои особенности имеет и ток при параллельном соединении резисторов. Попадая в первый узел соединения, он разделяется на столько частей, сколько имеется резисторов, подключенных параллельно. То есть, через сопротивление R1 будет протекать ток I1, а через R2 – ток I2. При попадании во второй узел, они вновь соединяются в один общий ток: I = I1 + I2.

Если какой-либо резистор вышел из строя, то остальные будут нормально функционировать. В этом заключается основное преимущество параллельного соединения. Особенно, это касается двигателей и электрических ламп, работающих от определенного номинального напряжения.

Расчет общего номинального сопротивления осуществляется с помощью формулы: R(общ)=1/(1/R1+1/R2+1/R3+1/R n), где R(общ) – является общим сопротивлением, а R1, R2, R3 и Rn – параллельно подключенными резисторами. Если выполняется параллельное соединение двух резисторов, при котором используется всего лишь два элемента, то в этом случае для расчетов используется следующая схема: R(общ)=R1хR2/R1+R2.

Очень часто в радиоэлектронике приходится пользоваться следующим правилом: если резисторы, подключенные параллельно, имеют один и тот же номинал, то итоговое сопротивление высчитывается путем деления номинала на число подключенных элементов. Такое параллельное соединение резисторов формула представляется следующим образом: R(общ)=R1\n, где R(общ) представляет собой сопротивление, R – номинал параллельно подключенного резистора, n – число подключенных элементов.

Для того чтобы рассчитать параллельное соединение резисторов, следует учитывать, что итоговое сопротивление всех подключенных элементов будет всегда ниже, чем сопротивление резистора с самым низким номиналом. В качестве примера можно рассмотреть схему с тремя резисторами, сопротивления которых составляют 30, 100 и 150 Ом. При использовании основной формулы будет получен следующий результат: R(общ)=1/(1/30+1/100+1/150) =1/(0,03+0,01+0,007)=1/0,047=21,28Ом. Таким образом, три резистора, соединенные параллельно, с минимальным номиналом 30 Ом, в итоге дадут общее сопротивление электрической цепи 21,28 Ом.

Онлайн калькулятор

В случае больших объемов вычислений, расчет параллельного соединения резисторов выполняется с помощью онлайн-калькулятора.

Соединение резисторов. Типы соединений и формулы расчёта общего сопротивления резисторов.

Как правильно соединять резисторы?

О том, как соединять конденсаторы и рассчитывать их общую ёмкость уже рассказывалось на страницах сайта. А как соединять резисторы и посчитать их общее сопротивление? Именно об этом и будет рассказано в этой статье.

Резисторы есть в любой электронной схеме, причём их номинальное сопротивление может отличаться не в 2 – 3 раза, а в десятки и сотни раз. Так в схеме можно найти резистор на 1 Ом, и тут же неподалёку на 1000 Ом (1 кОм)!

Поэтому при сборке схемы либо ремонте электронного прибора может потребоваться резистор с определённым номинальным сопротивлением, а под рукой такого нет. В результате быстро найти подходящий резистор с нужным номиналом не всегда удаётся. Это обстоятельство тормозит процесс сборки схемы или ремонта. Выходом из такой ситуации может быть применение составного резистора.

Для того чтобы собрать составной резистор нужно соединить несколько резисторов параллельно или последовательно и тем самым получить нужное нам номинальное сопротивление. На практике это пригождается постоянно. Знания о правильном соединении резисторов и расчёте их общего сопротивления выручают и ремонтников, восстанавливающих неисправную электронику, и радиолюбителей, занятых сборкой своего электронного устройства.

Последовательное соединение резисторов.

В жизни последовательное соединение резисторов имеет вид:


Последовательно соединённые резисторы серии МЛТ

Принципиальная схема последовательного соединения выглядит так:

На схеме видно, что мы заменяем один резистор на несколько, общее сопротивление которых равно тому, который нам необходим.

Подсчитать общее сопротивление при последовательном соединении очень просто. Нужно сложить все номинальные сопротивления резисторов входящих в эту цепь. Взгляните на формулу.

Общее номинальное сопротивление составного резистора обозначено как Rобщ.

Номинальные сопротивления резисторов включённых в цепь обозначаются как R1, R2, R3,…RN.

Применяя последовательное соединение, стоит помнить одно простое правило:

Из всех резисторов, соединённых последовательно главную роль играет тот, у которого самое большое сопротивление. Именно он в значительной степени влияет на общее сопротивление.

Что это значит?

Так, например, если мы соединяем три резистора, номинал которых равен 1, 10 и 100 Ом, то в результате мы получим составной на 111 Ом. Если убрать резистор на 100 Ом, то общее сопротивление цепочки резко уменьшиться до 11 Ом! А если убрать, к примеру, резистор на 10 Ом, то сопротивление будет уже 101 Ом. Как видим, резисторы с малыми сопротивлениями в последовательной цепи практически не влияют на общее сопротивление.

Параллельное соединение резисторов.

Можно соединять резисторы и параллельно:


Два резистора МЛТ-2, соединённых параллельно

Принципиальная схема параллельного соединения выглядит следующим образом:

Для того чтобы подсчитать общее сопротивление нескольких параллельно соединённых резисторов понадобиться знание формулы. Выглядит она вот так:

Эту формулу можно существенно упростить, если применять только два резистора. В таком случае формула примет вид:

Есть несколько простых правил, позволяющих без предварительного расчёта узнать, каково должно быть сопротивление двух резисторов, чтобы при их параллельном соединении получить то, которое требуется.

Если параллельно соединены два резистора с одинаковым сопротивлением, то общее сопротивление этих резисторов будет ровно в два раза меньше, чем сопротивление каждого из резисторов, входящих в эту цепочку.

Это правило исходит из простой формулы для расчёта общего сопротивления параллельной цепи, состоящей из резисторов одного номинала. Она очень проста. Нужно разделить номинальное сопротивление одного из резисторов на общее их количество:

Здесь R1 – номинальное сопротивление резистора. N – количество резисторов с одинаковым номинальным сопротивлением.

Ознакомившись с приведёнными формулами, вы скажите, что все они справедливы для расчёта ёмкости параллельно и последовательно соединённых конденсаторов. Да, только в отношении конденсаторов всё действует с точностью до «наоборот”. Узнать подробнее о соединении конденсаторов можно здесь.

Проверим справедливость показанных здесь формул на простом эксперименте.

Возьмём два резистора МЛТ-2 на 3 и 47 Ом и соединим их последовательно. Затем измерим общее сопротивление получившейся цепи цифровым мультиметром. Как видим оно равно сумме сопротивлений резисторов, входящих в эту цепочку.


Замер общего сопротивления при последовательном соединении

Теперь соединим наши резисторы параллельно и замерим их общее сопротивление.


Измерение сопротивления при параллельном соединении

Как видим, результирующее сопротивление (2,9 Ом) меньше самого меньшего (3 Ом), входящего в цепочку. Отсюда вытекает ещё одно известное правило, которое можно применять на практике:

При параллельном соединении резисторов общее сопротивление цепи будет меньше наименьшего сопротивления, входящего в эту цепь.

Что ещё нужно учитывать при соединении резисторов?

Во-первых, обязательно учитывается их номинальная мощность. Например, нам нужно подобрать замену резистору на 100 Ом и мощностью 1 Вт. Возьмём два резистора по 50 Ом каждый и соединим их последовательно. На какую мощность рассеяния должны быть рассчитаны эти два резистора?

Поскольку через последовательно соединённые резисторы течёт один и тот же постоянный ток (допустим 0,1 А), а сопротивление каждого из них равно 50 Ом, тогда мощность рассеивания каждого из них должна быть не менее 0,5 Вт. В результате на каждом из них выделится по 0,5 Вт мощности. В сумме это и будет тот самый 1 Вт.

Данный пример достаточно грубоват. Поэтому, если есть сомнения, стоит брать резисторы с запасом по мощности.

Подробнее о мощности рассеивания резистора читайте тут.

Во-вторых, при соединении стоит использовать однотипные резисторы, например, серии МЛТ. Конечно, нет ничего плохого в том, чтобы брать разные. Это лишь рекомендация.

Главная &raquo Радиоэлектроника для начинающих &raquo Текущая страница

Также Вам будет интересно узнать:

 

Параллельное соединение резисторов | Электротехника

Параллельное соединение резисторов. При параллельном соединении резисторов нескольких приемников они включаются между двумя точками электрической цепи, образуя параллельные ветви (рис. 26, а). Заменяя

Рис. 26. Схемы параллельного соединения приемников

лампы резисторами с сопротивлениями R1, R2, R3, получим схему, показанную на рис. 26, б.
При параллельном соединении ко всем резисторам приложено одинаковое напряжение U. Поэтому согласно закону Ома:

I1=U/R1; I2=U/R2; I3=U/R3.

Ток в неразветвленной части цепи согласно первому закону Кирхгофа I = I1+I2+I3, или

I = U / R1 + U / R2 + U / R3 = U (1/R1 + 1/R2 + 1/R3) = U / Rэк (23)

Следовательно, эквивалентное сопротивление рассматриваемой цепи при параллельном соединении трех резисторов определяется формулой

1/Rэк = 1/R1 + 1/R2 + 1/R3 (24)

Вводя в формулу (24) вместо значений 1/Rэк, 1/R1, 1/R2 и 1/R3 соответствующие проводимости Gэк, G1, G2 и G3, получим: эквивалентная проводимость параллельной цепи равна сумме проводимостей параллельно соединенных резисторов:

Gэк = G1+ G2 +G3 (25)

Таким образом, при увеличении числа параллельно включаемых резисторов результирующая проводимость электрической цепи увеличивается, а результирующее сопротивление уменьшается.
Из приведенных формул следует, что токи распределяются между параллельными ветвями обратно пропорционально их электрическим сопротивлениям или прямо пропорционально их проводимостям. Например, при трех ветвях

I1 : I2 : I3 = 1/R1 : 1/R2 : 1/R3 = G1 + G2 + G3 (26)

В этом отношении имеет место полная аналогия между распределением токов по отдельным ветвям и распределением потоков воды по трубам.
Приведенные формулы дают возможность определить эквивалентное сопротивление цепи для различных конкретных случаев. Например, при двух параллельно включенных резисторах результирующее сопротивление цепи

Rэк=R1R2/(R1+R2)

при трех параллельно включенных резисторах

Rэк=R1R2R3/(R1R2+R2R3+R1R3)

При параллельном соединении нескольких, например n, резисторов с одинаковым сопротивлением R1 результирующее сопротивление цепи Rэк будет в n раз меньше сопротивления R1, т.е.

Rэк = R1 / n (27)

Проходящий по каждой ветви ток I1, в этом случае будет в п раз меньше общего тока:

I1 = I / n (28)

При параллельном соединении приемников, все они находятся под одним и тем же напряжением, и режим работы каждого из них не зависит от остальных. Это означает, что ток, проходящий по какому-либо из приемников, не будет оказывать существенного влияния на другие приемники. При всяком выключении или выходе из строя любого приемника остальные приемники остаются включенными. Поэтому параллельное соединение имеет существенные преимущества перед последовательным, вследствие чего оно получило наиболее широкое распространение. В частности, электрические лампы и двигатели, предназначенные для работы при определенном (номинальном) напряжении, всегда включают параллельно.
На электровозах постоянного тока и некоторых тепловозах тяговые двигатели в процессе регулирования скорости движения нужно включать под различные напряжения, поэтому они в процессе разгона переключаются с последовательного соединения на параллельное.

Формула расчета сопротивления при параллельном соединении резистора

Параллельное соединение резисторов — одно из двух видов электрических соединений, когда оба вывода одного резистора соединены с соответствующими выводами другого резистора или резисторов. Зачастую резисторы соединяют последовательно или параллельно для того, чтобы создать более сложные электронные схемы.

Схема параллельного соединения резисторов показан на рисунке ниже. При параллельном соединении резисторов, напряжение на всех резисторах будет одинаковым, а протекающий через них ток будет пропорционален их сопротивлению:

Формула параллельного соединения резисторов

Общее сопротивление нескольких резисторов соединенных параллельно определяется по следующей формуле:

Ток, протекающий через отдельно взятый резистор, согласно закону Ома, можно найти по формуле:

При разработке устройства, возникла необходимость установить резистор с сопротивлением 8 Ом. Если мы просмотрим весь номинальный ряд стандартных значений резисторов, то мы увидим, что резистора с сопротивлением в 8 Ом в нем нет.

Выходом из данной ситуации будет использование двух параллельно соединенных резисторов. Эквивалентное значение сопротивления для двух резисторов соединенных параллельно рассчитывается следующим образом:

Данное уравнение показывает, что если R1 равен R2, то сопротивление R составляет половину сопротивления одного из двух резисторов. При R = 8 Ом, R1 и R2 должны, следовательно, иметь значение 2 × 8 = 16 Ом.
Теперь проведем проверку, рассчитав общее сопротивление двух резисторов:

Таким образом, мы получили необходимое сопротивление 8 Ом, соединив параллельно два резистора по 16 Ом.

Пример расчета №2

Найти общее сопротивление  R из трех параллельно соединенных резисторов:

Общее сопротивление R рассчитывается по формуле:

Этот метод расчета может быть использованы для расчета любого количества отдельных сопротивлений соединенных параллельно.

Один важный момент, который необходимо запомнить при расчете параллельно соединенных резисторов – это то, что общее сопротивление всегда будет меньше, чем значение наименьшего сопротивления в этой комбинации.

Как рассчитать сложные схемы соединения резисторов

Более сложные соединения резисторов могут быть рассчитаны путем систематической группировки резисторов. На рисунке ниже необходимо посчитать общее сопротивление цепи, состоящей из трех резисторов:

Резисторы R2 и R3 соединены последовательно (группа 2). Они в свою очередь соединены параллельно с резистором R1 (группа 1).

Последовательное соединение резисторов группы 2 вычисляется как сумма сопротивлений R2 и R3:

В результате мы упрощаем схему в виде двух параллельных резисторов. Теперь общее сопротивление всей схемы можно посчитать следующим образом:

Расчет более сложных соединений резисторов можно выполнить используя законы Кирхгофа.

Ток, протекающий в цепи параллельно соединенных резисторах

Общий ток I протекающий в цепи параллельных резисторов равняется сумме отдельных токов, протекающих во всех параллельных ветвях, причем ток в отдельно взятой ветви не обязательно должен быть равен току в соседних ветвях.

Несмотря на параллельное соединение, к каждому резистору приложено одно и то же напряжение. А поскольку величина сопротивлений в параллельной цепи может быть разной, то и величина протекающего тока через каждый резистор тоже будет отличаться (по определению закона Ома).

Рассмотрим это на примере двух параллельно соединенных резисторов. Ток, который течет через каждый из резисторов ( I1 и I2 ) будет отличаться друг от друга поскольку сопротивления резисторов R1 и R2 не равны.
Однако мы знаем, что ток, который поступает в цепь в точке «А» должен выйти из цепи в точке «B» .

Первое правило Кирхгофа гласит: «Общий ток, выходящий из цепи равен току входящий в цепь».

  • Таким образом, протекающий общий ток в цепи  можно определить как:
  • I = I1 + I2
  • Затем с помощью закона Ома можно вычислить ток, который протекает через каждый резистор:
  • Ток, протекающий в R1 = U ÷ R1 = 12 ÷ 22 кОм = 0,545 мА
  • Ток, протекающий в R 2 = U ÷ R2 = 12 ÷ 47 кОм = 0,255 мА
  • Таким образом, общий ток будет равен:
  • I = 0,545 мА + 0,255 мА = 0,8 мА
  • Это также можно проверить, используя закон Ома:
  • I = U ÷ R = 12 В ÷ 15 кОм = 0,8 мА (то же самое)
  • где 15кОм — это общее сопротивление двух параллельно соединенных резисторов (22 кОм и 47 кОм)
  • И в завершении хочется отметить, что большинство современных резисторов маркируются цветными полосками и назначение ее можно узнать здесь.

Параллельное соединение резисторов — онлайн калькулятор

Чтобы быстро вычислить общее сопротивление двух и более резисторов, соединенных параллельно, вы можете воспользоваться следующим онлайн калькулятором:

Подведем итог

Когда два или более резистора соединены так, что оба вывода одного резистора соединены с соответствующими выводами другого резистора или резисторов, то говорят, что они соединены между собой параллельно. Напряжение на каждом резисторе внутри параллельной комбинации одинаковое, но токи, протекающие через них, могут отличаться друг от друга, в зависимости от величины сопротивлений каждого резистора.

Эквивалентное или полное сопротивление параллельной комбинации всегда будет меньше минимального сопротивления резистора, входящего в параллельное соединение.

Источник: http://www.joyta.ru/7362-parallelnoe-soedinenie-rezistorov/

Последовательное и параллельное соединение резисторов

Последовательное соединение – это соединение двух или более резисторов в форме цепи, в которой каждый отдельный резистор соединяется с другим отдельным резистором только в одной точке.

Общее сопротивление Rобщ

При таком соединении, через все резисторы проходит один и тот же электрический ток. Чем больше элементов на данном участке электрической цепи, тем «труднее» току протекать через него. Следовательно, при последовательном соединении резисторов их общее сопротивление увеличивается, и оно равно сумме всех сопротивлений.

Напряжение при последовательном соединении

Напряжение при последовательном соединении распределяется на каждый резистор согласно закону Ома:

Т.е чем большее сопротивление резистора, тем большее напряжение на него падает.

Параллельное соединение резисторов

Параллельное соединение – это соединение, при котором резисторы соединяются между собой обоими контактами. В результате к одной точке (электрическому узлу) может быть присоединено несколько резисторов.

Общее сопротивление Rобщ

При таком соединении, через каждый резистор потечет отдельный ток. Сила данного тока будет обратно пропорциональна сопротивлению резистора. В результате общая проводимость такого участка электрической цепи увеличивается, а общее сопротивление в свою очередь уменьшается.

Таким образом, при параллельном подсоединении резисторов с разным сопротивлением, общее сопротивление будет всегда меньше значения самого маленького отдельного резистора.

Формула общей проводимости при параллельном соединении резисторов:

Формула эквивалентного общего сопротивления при параллельном соединении резисторов:

Для двух одинаковых резисторов общее сопротивление будет равно половине одного отдельного резистора:

Соответственно, для n одинаковых резисторов общее сопротивление будет равно значению одного резистора, разделенного на n.

Напряжение при параллельном соединении

Напряжение между точками A и B является как общим напряжением для всего участка цепи, так и напряжением, падающим на каждый резистор в отдельности. Поэтому при параллельном соединении на все резисторы упадет одинаковое напряжение.

Электрический ток при параллельном соединении

Через каждый резистор течет ток, сила которого обратно пропорциональна сопротивлению резистора. Для того чтобы узнать какой ток течет через определенный резистор, можно воспользоваться законом Ома:

Смешанное соединение резисторов

Смешанным соединением называют участок цепи, где часть резисторов соединяются между собой последовательно, а часть параллельно. В свою очередь, смешанное соединение бывает последовательного и параллельного типов.

Общее сопротивление Rобщ

Для того чтобы посчитать общее сопротивление смешанного соединения:

  • Цепь разбивают на участки с только пареллельным или только последовательным соединением.
  • Вычисляют общее сопротивление для каждого отдельного участка.
  • Вычисляют общее сопротивление для всей цепи смешанного соединения.

Так это будет выглядеть для схемы 1:

Также существует более быстрый способ расчета общего сопротивления для смешанного соединения. Можно, в соответствии схеме, сразу записывать формулу следующим образом:

  • Если резисторы соединяются последоватеьно — складывать.
  • Если резисторы соединяются параллельно — использовать условное обозначение «||».
  • Подставлять формулу для параллельного соединения где стоит символ «||».

Так это будет выглядеть для схемы 1:

После подстановки формулы параллельного соединения вместо «||»:

Источник: http://hightolow.ru/resistor3.php

Параллельное соединение сопротивлений в электрической цепи. Параллельное соединение конденсаторов и катушек

Параллельное соединение электрических элементов (проводников, сопротивлений, емкостей, индуктивностей) — это такое соединение, при котором подключенные элементы цепи имеют два общих узла подключения.

Другое определение: сопротивления подключены параллельно, если они подключены одно и той же паре узлов. 

Графическое обозначение схемы параллельного соеднинения

На приведенном рисунке показана схема параллельное подключения сопротивлений R1, R2, R3, R4. Из схемы видно, что все эти четыре сопротивления имеют две общие точки (узла подключения). 

В электротехнике принято, но не строго требуется, рисовать провода горизонтально и вертикально. Поэтому эту же схему можно изобразить, как на рисунке ниже. Это тоже параллельное соединение тех же самых сопротивлений.

Формула для расчета параллельного соединения сопротивлений

При параллельном соединении обратная величина от эквивалентного сопротивления равна сумме обратных величин всех параллельно подключенных сопротивлений. Эквивалентная проводимость равна сумме всех параллельно подключенных проводимостей электрической схемы.

Для приведенной выше схемы эквивалентное сопротивление можно рассчитать по формуле:

В частном случае при подключении параллельно двух сопротивлений:

Эквивалентное сопротивление цепи определяется по формуле:

 В случае подключения «n» одинаковых сопротивлений, эквивалентное сопротивление можно рассчитать по частной формуле:

Формулы для частного рассчета вытекают из основной формулы. 

Формула для расчета параллельного соединения емкостей (конденсаторов)

При параллельном подключении емкостей (конденсаторов) эквивалентная емкость равна сумме параллельно подключенных емкостей:

 

Формула для расчета параллельного соединения индуктивностей

  • При параллельном подключении индуктивностей, эквивалентная индуктивность рассчитывается так же, как и эквивалентное сопротивление при параллельном соединении: 
  •  
  • Необходимо обратить внимание, что в формуле не учтены взаимные индуктивности.

Пример свертывания параллельного сопротивления  

Для участка электрической цепи необходимо найти параллельное соединение сопротивлений выполнить их преобразование до одного.

Из схемы видно, что параллельно подключены только R2 и R4. R3 не параллельно, т.к. одним концом оно подключено к источнику ЭДС E1. R1 — одним концом подключено к R5, а не к узлу. R5 — одним концом подключено к R1, а не к узлу. Можно так же говорить, что последовательное соединение сопротивлений R1 и R5 подключено параллельно с R2 и R4.

Рассчитать эквивалентное сопротивлений R14 можно по формуле для двух сопротивлений.

Ток при параллельном соединении

При параллельном соединении сопротивлений ток через каждое сопротивление в общем случае разный. Величина тока обратно пропорциональна величине сопротивления.

Напряжение при параллельном соединении 

При параллельном соединении разность потенциалов между узлами, объединяющими элементы цепи, одинакова для всех элементов.

Применение параллельного соединения

1. В промышленности изготавливаются сопротивления определенных величин. Иногда необходимо получить значение сопротивления вне данных рядов. Для этого можно подключить несколько сопротивлений параллельно. Эквивалентное сопротивление всегда будет меньше самого большого номинала сопротивления.

2. Делитель токов.

Источник: https://kurstoe.ru/osnovnie-svedeniya/preobrazovanie-tcepej/parallelnoe-soedinenie.html

Последовательное и параллельное соединение резисторов

Последовательное и параллельное соединение резисторов в схемах являются самыми распространенными, также — это база для расчета более сложных схем.

Последовательное подключение

Начнем с последовательного соединения. По этой схеме каждый резистор подключается с другим только в одной точке, их может быть в цепи 2, 3 и больше.

Рис. Последовательное подключение.

Обозначение:

Обозначим сопротивления: R1, R2, R3 и напряжение источника в цепи Uц. При подключении источника питания в ней начнет протекать ток Iц. В цепи с последовательным соединением ток протекает по всем резисторам один за другим.

Поскольку ток течет через все резисторы их сопротивления и ток суммируется, Iц = I1+I2+I3, Rц = R1 +R2 + R3, чем больше отдельно взятое сопротивление, тем тяжелее электронам преодолевать участок цепи. Мощность резисторов при последовательном и параллельном соединении рассчитывается по разным формулам.

В последовательных цепях — складываем, в параллельных — это обратно пропорциональная величина.

Параллельное соединение

Рис. Параллельное подключение.

Данный вид подключения характерен тем, что все элементы цепи соединяется выводами в одной точке друг другу, т.е. точка входа и выхода всех нагрузок сходятся в одну точку (или еще одно обозначение на схемах — //). Электроток, двигаясь по проводнику, дойдя до общего соединения делится на количество имеющихся веток. Если представить движение воды в трубе, то можно сказать, что вода двигающиеся по одной трубе, равномерно перетекает в несколько отводов, подсоединенных к ней. В нашем случае заряженные электроны, двигающиеся по проводнику, также растекаются на количества предложенных веток в узле. Более наглядно это можно представить в виде формул: 1. Каждый вид соединения находится под одинаковым напряжением: U = U1 = U2; 2. Суммарная сила тока равняется суммарному значению тока каждого участка I = I1 + I2; 3. Сопротивление цепи равно сумме величина обратных сопротивлению участка: 1/R = 1/R1 + 17R2 + . . . + 1/Rn; 4. Сила тока пропорциональна сопротивлению каждого участка I1/I2=R2/R1.

Далее рассмотрим схему как работает не только последовательное параллельное, но и смешанное соединение резисторов.

Смешанное подключение

Рис. Смешанное подключение резисторов

В электрических схемах используются не только типовые схемы, но и смешанное, созданное из критерий определенных требований. Чаще всего в схемах встречается третий вариант, представляющий набор из элементарных типов схем. В смешанных участках учитываются не только элементы, но и направления движения тока. При вычислении мощности резисторов смешанного подключения используются формулы для параллельного и последовательного соединения резисторов, формула также является составной.

Основные законы электротехники, наиболее часто используемые для расчетов

Рассмотрим основные законы электротехники и свойства последовательного и параллельного соединения резисторов для участка цепи

Закон Ома

Напряжение находится по закону Ома по формуле I=U/R — чем больше сопротивление, тем меньше ток. Напряжение можно найти из этой же формулы. U=R*I, ток умножается на сопротивление. Запишем эту формулу для каждого участка U1=R1· I1, Un=Rn · In.

Законы Кирхгофа

Первый закон

Ещё один очень важный закон — это закон Кирхгофа. Для участка цепи постоянного тока их два.

Рис. иллюстрация к пояснению действия первого закона Кирхгофа.

Первый закон имеет формулировку: Сумма всех токов, входящих в узел и выходящих из него равна нулю. Если посмотреть на схему, I1 — это ток, который заходит в узел, I2 и I3 — это электроны, которые вытекают из него. Применяя формулировку первого закона можно записать формулу по-другому:

I1-I2+I3=0. В этой формуле знаки плюс имеют значения, которые прибывают в узел, минус, который отходит от него.

Второй закон Кирхгофа

Рис. иллюстрация к пояснению действия второго закона Кирхгофа.

Если к цепи с включенными сопротивлениями подключен один источник ЭДС (батарея питания) тогда всё понятно, можно обойтись законом Ома. А, если, источников несколько и схема с различным схемным расположением элементов, тогда вступает в силу второй закон, который гласит: сумма токов всех источников питания для замкнутого контура, равна сумме падений напряжения на всех сопротивлениях участка в этом контуре. E1- Е2 = — UR1 — UR2 или E1 = Е2 — UR1 — UR2.

Параллельное и последовательное соединение резисторов,  решение задач

Алгоритм расчёта смешанных подключений находится в тех же правилах, что и в элементарных схемах расчета последовательного и параллельного соединения резисторов. Ничего нового нет: нужно правильно разбить предложенную схему на пригодные для расчета участки. Участки, с элементами, подключены поочередно либо параллельно.

Рис. Порядок замещения при расчете сложных позиций более простыми.

Для решения задачи на последовательное и параллельное соединение резисторов необходимо правильно оценить цепи элементов. Рассмотрим схему №1 на рис.

На схеме присутствует параллельная и последовательная часть соединения элементов. Для расчета очень важно аккуратно, шаг за шагом упрощать цепи и не брать сразу всю схему (рис.1).

Как же правильно определить параллельное и последовательное соединение резисторов?

Для примера расчета возьмем резисторы R3, R4, которые подключены параллельно. Эквивалентный резистор этих элементов, будет равенRэ. = 1/R34 =1/R3 + 1/R4, после преобразования формулы и приведения к одному знаменателю получим R34 = R3 · R4 / (R3 + R4). Э. = 1/3+1/4 /(3+4) =1,7 Ом.

Далее видно, что приведённая эквивалентное R эк и R6 соединены последовательно, чтобы узнать сопротивление их необходимо сложить, тогда общее сопротивление будет равно R346 = R34 + R6, тогда Rэк346 = 1,7 + 6 = 7, 7 Ом. Заменяем на схеме одним общим элементом, теперь, позиция упрощается еще больше (рис 3).

Теперь образовалась ситуация — включение трех элементов в //. Как вычисляется такое соединение нам уже известно, 1/ R23465 = 1/ R2 +1/R346 + 1/R5 после вычисления правой части получаем 0,82 Ом. После окончательного вычисления получаем R23465 = 2,1 Ом. Здесь следует обратить внимание, что общее сопротивление получилось меньше самого меньшего из трех.

Заменяем эти сопротивление одним эквивалентным R23465. В конечном итоге все выглядит уже намного проще. Rц = Rэк + R1+ R2. R об. = R ц = 1,21 +7+1 =9,21 Ом. Из приведенного алгоритма расчёта видно, как из сложной схемы путем простого математического вычисления и применения правил сокращения резисторов участок становится простой и понятной.

Схема с подключением сопротивлений «треугольником»

Рис. Расчетная схема соединения резисторов в треугольник.

Иногда некоторые затруднения возникают при разборе схемы соединения в треугольник.

Рассмотрим на примере рисунка расчет резисторов по этому подключению. Из схемы видно, что R1 и R2 соединены последовательно Rэ12 будет соединяться R3 последовательно.

Затем Rэ123 соединяется с сопротивлением R4, R5 в последовательную цепь. Затем все это объединяется с Rэ в //.

Проведем несложные вычисления учитывая, что R1, R2, R4, R5 равняется 1 Ом. R3, R7 — 2 Ом.

RЭ1,2 = R1+R2 = 1+1=2 Ом.

Вычисляем параллельное подключение: Rэ 12 с R3. Rэ1,3 = (Rэ12*R3) /(Rэ12+R3) = (2*2) /(2+2) = 1Ом.

Далее мы видим последовательное: RЭ123 + R4 + R5 = 1+1+1 = 3 Ом. И последнее — Rэ123 4 5 с R6 — параллельное.

Общее сопротивление цепи Rц = Rоб = (RЭ1,2,3,4,5 *R6) /(RЭ1,2,3,4,5+R6) = (3 * 2) / (3+2) = 1,2 Ом. Как видно, что расчет подобного варианта также не сложный.

Расчет последовательного и параллельного подключения резисторов онлайн

Подсчитать значение мощность и сопротивлений подставляя их в формулы можно только в учебных целях, или, когда объемы не очень большие.

Наиболее практичный вариант расчета является онлайн калькуляторы, которые расположены на многочисленных интернет ресурсах.

Для расчёта любой сложности нужно правильно определить тип соединения резисторов последовательное или параллельное и внести данные для расчета в поля калькулятора.

Также такая форма расчета подойдет и для проверки результатов решения учебных задач.

Последовательное и параллельное соединение резисторов и конденсаторов

Электрические цепи состоят не только из резисторов, в них применяется большое количество различных деталей, например, конденсатор, которые подключаются в последовательное, // и смешанное соединение.

Рис. Замещения последовательно включенных элементов.

Определение этому элементу можно дать следующее: Конденсатор — это совокупность проводящих тел служащий для накопления электрического заряда. Элементарный конденсатор имеет две пластины, форма этих пластин может быть различной: сферической, круглой, цилиндрической, прямоугольной — по форме пластин разделяется и тип конденсатора.

Важное свойство. Одно из важных свойств конденсатора: если заряжается одна пластина конденсатора, то благодаря явлению электростатической индукции заряжается и вторая половина, но с противоположным знаком.

Устройство конденсатора

Плоский конденсатор состоит из двух плоских пластин отстоящих друг от друга на маленькое расстояние. У конденсатора к двум пластинам припаивается вывод всего их получается два.

Типовые схемы подключения конденсаторов

Рассмотрим различные виды подключения конденсатора.

Последовательное

Первый вид — это последовательное соединение. Предположим, что емкость этих конденсаторов будут равны.

Тогда заряды также будут равны: q1=q2=q3, как и в примере с резисторами,  сложный тип позиций с конденсатором можно упростить, заменив несколько элементов одним.

У элементов соединенных друг за другом, общая емкость будет обратно пропорциональная всем имеющимся элементам. То есть: Rэк будет равняться 1/С1 + 1/С2 +…. 1/Сn/

Напряжение складывается,  U эк = U1 + U2+ … Un.

Параллельное

Второй тип подключения конденсаторов — это соединение в паралель

Рис. Схема замещения элементов, включенных в параллель.

  • Соответственно эти конденсаторов обозначены C1, C2, … Cn заряды: Q1, Q2, … Qn и напряжение: U1, U2, … Un.
  • У элементов в // емкость складывается Сэ = C1 + C2 + … C n. Напряжение Un на каждом конденсаторе будет равно напряжению на эквивалентном
  • Uэ = U1 = U2 =… = Un — это особенность параллельного подсоединения всех элементов цепи.
  • Емкость будет складываться из суммы отдельных элементов Сэ =С1 + С2 + … Сп.

Рис. Расчетные позиции элементов при различном включении.

Простая позиция, которая не требует преобразования №1 — последовательное подключение. По известной формуле для этих поз. запишем 1/Сэ = 1/С1 +1/С2 +1/С3,  подставив формулу значения, которые даны в условии задачи, получим 1/Сэ = 1/С1 +1/С2 +1/С3 = 59 мФ.

Не требует преобразования и 2 схема: емкость общего конденсатора будет равняться сумме конденсаторов которые включены в параллельной цепи: Сэ =С1 +С2 +С3 Сэ = 100 + 200 + 500 = 800 мФ.

Рассмотрев рис. №3 видно, что пара конденсаторов включена параллельно и один последовательно. Алгоритм преобразования таких цепей мы уже рассматривали, поэтому: сразу же находим емкость конденсатора Сэ соединения: Сэ = С1+С2 = 200+500 = 700 мФ.

Теперь находим общие эквивалентную емкость элементов с последовательным подключением 1/Сэ = 1/С2,3 +1/ С1 = 89 мф. Практическая задача решена.

Источник: http://themechanic.ru/posledovatelnoe-i-parallelnoe-soedinenie-rezistorov/

Соединение резисторов

Радиоэлектроника для начинающих

О том, как соединять конденсаторы и рассчитывать их общую ёмкость уже рассказывалось на страницах сайта. А как соединять резисторы и посчитать их общее сопротивление? Именно об этом и будет рассказано в этой статье.

Резисторы есть в любой электронной схеме, причём их номинальное сопротивление может отличаться не в 2 – 3 раза, а в десятки и сотни раз. Так в схеме можно найти резистор на 1 Ом, и тут же неподалёку на 1000 Ом (1 кОм)!

Поэтому при сборке схемы либо ремонте электронного прибора может потребоваться резистор с определённым номинальным сопротивлением, а под рукой такого нет. В результате быстро найти подходящий резистор с нужным номиналом не всегда удаётся. Это обстоятельство тормозит процесс сборки схемы или ремонта. Выходом из такой ситуации может быть применение составного резистора.

Для того чтобы собрать составной резистор нужно соединить несколько резисторов параллельно или последовательно и тем самым получить нужное нам номинальное сопротивление. На практике это пригождается постоянно.

Знания о правильном соединении резисторов и расчёте их общего сопротивления выручают и ремонтников, восстанавливающих неисправную электронику, и радиолюбителей, занятых сборкой своего электронного устройства.

Последовательное соединение резисторов

  • В жизни последовательное соединение резисторов имеет вид:
  • Последовательно соединённые резисторы серии МЛТ
  • Принципиальная схема последовательного соединения выглядит так:

На схеме видно, что мы заменяем один резистор на несколько, общее сопротивление которых равно тому, который нам необходим.

Подсчитать общее сопротивление при последовательном соединении очень просто. Нужно сложить все номинальные сопротивления резисторов входящих в эту цепь. Взгляните на формулу.

  1. Общее номинальное сопротивление составного резистора обозначено как Rобщ.
  2. Номинальные сопротивления резисторов включённых в цепь обозначаются как R1, R2, R3,…RN.
  3. Применяя последовательное соединение, стоит помнить одно простое правило:

Из всех резисторов, соединённых последовательно главную роль играет тот, у которого самое большое сопротивление. Именно он в значительной степени влияет на общее сопротивление.

Что это значит?

Так, например, если мы соединяем три резистора, номинал которых равен 1, 10 и 100 Ом, то в результате мы получим составной на 111 Ом.

Если убрать резистор на 100 Ом, то общее сопротивление цепочки резко уменьшиться до 11 Ом! А если убрать, к примеру, резистор на 10 Ом, то сопротивление будет уже 101 Ом.

Как видим, резисторы с малыми сопротивлениями в последовательной цепи практически не влияют на общее сопротивление.

Параллельное соединение резисторов

  • Можно соединять резисторы и параллельно:
  • Два резистора МЛТ-2, соединённых параллельно
  • Принципиальная схема параллельного соединения выглядит следующим образом:

Для того чтобы подсчитать общее сопротивление нескольких параллельно соединённых резисторов понадобиться знание формулы. Выглядит она вот так:

Эту формулу можно существенно упростить, если применять только два резистора. В таком случае формула примет вид:

Есть несколько простых правил, позволяющих без предварительного расчёта узнать, каково должно быть сопротивление двух резисторов, чтобы при их параллельном соединении получить то, которое требуется.

Если параллельно соединены два резистора с одинаковым сопротивлением, то общее сопротивление этих резисторов будет ровно в два раза меньше, чем сопротивление каждого из резисторов, входящих в эту цепочку.

Это правило исходит из простой формулы для расчёта общего сопротивления параллельной цепи, состоящей из резисторов одного номинала. Она очень проста. Нужно разделить номинальное сопротивление одного из резисторов на общее их количество:

Здесь R1 – номинальное сопротивление резистора. N – количество резисторов с одинаковым номинальным сопротивлением.

Ознакомившись с приведёнными формулами, вы скажите, что все они справедливы для расчёта ёмкости параллельно и последовательно соединённых конденсаторов. Да, только в отношении конденсаторов всё действует с точностью до «наоборот”. Узнать подробнее о соединении конденсаторов можно здесь.

Проверим справедливость показанных здесь формул на простом эксперименте.

Возьмём два резистора МЛТ-2 на 3 и 47 Ом и соединим их последовательно. Затем измерим общее сопротивление получившейся цепи цифровым мультиметром. Как видим оно равно сумме сопротивлений резисторов, входящих в эту цепочку.

  1. Замер общего сопротивления при последовательном соединении
  2. Теперь соединим наши резисторы параллельно и замерим их общее сопротивление.
  3. Измерение сопротивления при параллельном соединении
  4. Как видим, результирующее сопротивление (2,9 Ом) меньше самого меньшего (3 Ом), входящего в цепочку. Отсюда вытекает ещё одно известное правило, которое можно применять на практике:

При параллельном соединении резисторов общее сопротивление цепи будет меньше наименьшего сопротивления, входящего в эту цепь.

Что ещё нужно учитывать при соединении резисторов?

Во-первых, обязательно учитывается их номинальная мощность. Например, нам нужно подобрать замену резистору на 100 Ом и мощностью 1 Вт. Возьмём два резистора по 50 Ом каждый и соединим их последовательно. На какую мощность рассеяния должны быть рассчитаны эти два резистора?

Поскольку через последовательно соединённые резисторы течёт один и тот же постоянный ток (допустим 0,1 А), а сопротивление каждого из них равно 50 Ом, тогда мощность рассеивания каждого из них должна быть не менее 0,5 Вт. В результате на каждом из них выделится по 0,5 Вт мощности. В сумме это и будет тот самый 1 Вт.

Данный пример достаточно грубоват. Поэтому, если есть сомнения, стоит брать резисторы с запасом по мощности.

Подробнее о мощности рассеивания резистора читайте тут.

Во-вторых, при соединении стоит использовать однотипные резисторы, например, серии МЛТ. Конечно, нет ничего плохого в том, чтобы брать разные. Это лишь рекомендация.

Главная » Радиоэлектроника для начинающих » Текущая страница

Также Вам будет интересно узнать:

Источник: https://go-radio.ru/connection-of-resistors.html

Параллельное соединение резисторов

Господа, в прошлый раз мы с вами говорили про последовательное сопротивление резисторов. Сегодня я бы хотел вам рассказать про другой возможный вид соединения – параллельное.

Чем различается последовательное и параллельное соединение я уже писал в предыдущей статье.  Но все-таки вытащу сюда картинку из той прошлой статьи, я ж знаю, что вам будет лень ходить по ссылкам .

  • А) – Последовательное соединение
  • В) – Параллельное соединение
  • Рисунок 1 – Последовательное и параллельное соединение
  • Как мы видим из рисунка 1, параллельное соединение – это такое соединение, при котором одни концы всех резисторов соединены в один узел, а другие концы – в другой узел.

Сейчас наша задача будет разобраться, как ведут себя токи, напряжения, сопротивления и мощности при таком подключении. Для этого прошу вас взглянуть на рисунок 2, где подробно разрисован расклад дел для параллельного соединения. Будем полагать, что мы знаем величины R1, R2 и R3, а также величину приложенного к схеме напряжения U. Про токи же мы ничего не знаем.

Рисунок 2 – Параллельное соединения

Что мы видим на рисунке 2? Ну, в первую очередь – два узла А и B. В узел А сходятся одни концы всех резисторов, а в узел В – другие концы. Пусть узел А имеет потенциал φ1, а узел В – потенциал φ2. Из рисунка 2 видно, что для всех резисторов R1, R2 и R3 у нас одна и та же разность потенциалов U.

Как следует из статьи про потенциалы, это означает, что напряжение на всех резисторах у нас одинаково и равно приложенному напряжению U. Это важный вывод, его следует хорошо запомнить.

С токами дело обстоит по-другому. Проанализируем рисунок 2 слева направо. Пусть у нас в цепи течет ток I. Течет он себе, течет, никого не трогает и тут вдруг натыкается на узел А. Что в этом случае говорит полюбившаяся вам статья про первый закон Кирхгофа? А то, что ток I в узле А разделится на три тока I1, I2, I3. При этом будет выполняться равенство

То есть через резистор R1 будет протекать ток I1, через резистор R2 – ток I2, а через резистор R3 – ток I3.

Итак, у нас в системе уже тихо-мирно текут себе три тока. И все хорошо, пока они не наткнуться на узел В. Тут снова вступает в силу первый закон Кирхгофа. Эти три тока I1, I2, I3 вновь соединятся в один ток I. Причем после узла В ток будет иметь такую же величину I, какой он был до узла А.

То есть если все вышесказанное воплотить в лаконичный язык наскальной живописи, положение дел можно представить себе вот так

Как же найти эти самые токи I1, I2, I3? Господа, полагаю, вы уже догадались, что на помощь нам придет горячо нами всеми любимый закон Ома. Действительно, мы знаем сопротивления резисторов и, кроме того, нам известно, что на всех них падает одно и тоже напряжение U. Поэтому легко находим токи

Отлично, мы разобрались с напряжениями и с токами в такой схеме. А помните в статье про последовательное сопротивление мы ловко преобразовали три резистора в один с эквивалентным им сопротивлением? Нельзя ли и здесь сделать что-то подобное? Оказывается, вполне себе можно. Как мы помним, токи в схеме распределены таким вот образом

  1. Обзовем эквивалентное сопротивление буковкой R. И подставим в это выражение только что найденные нами токи I1, I2, I3
  2. Видим, что здесь без проблем можно сократить левую и правую части на U. Получаем
  3. Господа, важный вывод: при параллельном соединении резисторов обратное эквивалентное сопротивление равно сумме обратных сопротивлений отдельных резисторов.
  4. То есть для упрощения различных расчетов электрических схем такую вот цепочку параллельно соединенных резисторов можно заменить одним резистором с соответствующим сопротивлением, как показано на рисунке 3.
  5. Рисунок 3 – Преобразование параллельного соединение

Весьма частый случай на практике, когда соединены параллельно не много резисторов, а всего два. Поэтому полезно знать наизусть итоговое сопротивление такой схемы. Давайте посмотрим, чему оно равно:

То есть, если у вас два сопротивления соединены параллельно, то по этой формуле вы легко высчитаете общее сопротивление. Рассмотрим пример. Пусть у нас параллельно соединены два резистора 10 кОм и 15 кОм. Чему равно их общее сопротивление?

Заметьте, господа, итоговое сопротивление у нас получилось 6 кОм, что меньше 10 кОм и 15 кОм. То есть при параллельном соединении общее сопротивление меньше любого из составляющих.

Это всегда верно для любого количества резисторов, а не только для двух. Итоговое сопротивление всегда уменьшается (в отличии от последовательного сопротивления, где итоговое сопротивление всегда растет).

Этот факт полезно запомнить.

Еще один часто встречающийся на практике случай – когда параллельно соединены несколько резисторов с одинаковым сопротивлением. Допустим, каждый из них обладает сопротивлением R1 и всего их N штук. Тогда по нашей общей формуле для эквивалентного сопротивления

  • То есть при параллельном соединении N одинаковых резисторов с сопротивлением R1 итоговое сопротивление будет в N раз меньше этого самого сопротивления R1.
  • Так-с, с током разобрались, с напряжением разобрались, с эквивалентным сопротивлением вроде тоже…осталась мощность. Для этого воспользуемся вот этим выражением, которое мы писали чуть выше в статье
  • Умножим левую и правую части на напряжение U.
  • Как мы помним из статьи про мощность произведение тока на напряжение есть мощность. То есть мы можем записать
  • где Р – мощность, выдаваемая источником;
  • P1 – мощность, рассеиваемая на резисторе R1;
  • P2 – мощность, рассеиваемая на резисторе R2;
  • P3 – мощность, рассеиваемая на резисторе R3.

Заметьте, господа, формула в точности такая же, как и для случая последовательного соединения резисторов. И там и там мощность, выдаваемая источником, равна сумме мощностей, рассеиваемых на резисторах цепи.

Итак, господа, мы рассмотрели основные соотношения при параллельном соединении резисторов. Теперь осталось поговорить, где это параллельное соединение можно использовать и для чего.

1) Ну, во-первых, параллельное соединение применяют во всех случаях, когда хотят запитать несколько нагрузок от одного источника напряжения. При этом пользуются тем свойством, что при параллельном соединении напряжения на всех нагрузках одинаково.

То есть, допустим, вы берете источник напряжения, выставляете на нем напряжение 5 В и цепляете к этому источнику сразу несколько своих устройств. Узлами А и В в этом случае будут клеммы источника. На каждое из устройств в этом случае придет напряжение 5 В.

Да и все устройства в вашей квартире (лампочки, компьютеры, телевизоры и все прочее) соединены между собой параллельно.

2) Второе возможное применение встречается не так часто, но, думаю, о нем тоже следует рассказать. Допустим, вы делаете какую-то схему, где необходим очень точный подгон сопротивления. Скажем, надо получить сопротивление 6 кОм. Такое сопротивление найти нелегко, их просто не продают. Зато у вас есть два сопротивления 10 кОм и 15 кОм.

Вы их соединяете параллельно и получаете требуемые 6 кОм. Как показывает практика, 3 параллельных резисторов достаточно для получения итогового результирующего сопротивления требуемого номинала с весьма хорошей точностью. Конечно, таких вещей лучше избегать и, если есть возможность, всегда стараться применять стандартные сопротивления.

Но бывают случаи, когда это невозможно, и тогда приходит на помощь этот метод.

3) Третий пункт будет немного похож на первый. Его суть заключается в следующим. Допустим, нам надо снять с источника питания 10 Вт мощности. А у нас в наличии только резисторы, которые позволяют рассеивать на себе 1 Вт. Что делать? Можно соединить 10 резисторов параллельно и с каждого снимать по 1 Вт. Мы же помним нашу формулу

Конечно, лучше брать не 10 резисторов, а хотя бы 15 и рассеивать на них меньше, чем 1 Вт. Работать на пределе никогда не следует.

Кстати, тут очень вовремя к моменту написания статьи пришли платы с производства! Господа, прошу вас взглянуть на рисунок 4.

Рисунок 4 – Плата нагревателя

На нем изображена плата нагревателя (флешка для масштаба). В чем суть? Имеется весьма сложное устройство, предназначенное для работы в арктических условиях.

Найти же компоненты, которые надежно функционировать при температурах минус 55 градусов и при этом стоят адекватных денег и обладают адекватными размерами бывает непросто. Обычно элементная база в лучшем случае рассчитана на минус 40 градусов.

И было принято решение разработать вот такой вот нагреватель для прогрева чувствительных к холоду аналоговых узлов устройства. Он управляется с микроконтроллера и автоматически включается при температурах меньше минус 40 градусов.

Как вы можете видеть из рисунка 4, этот нагреватель представляет собой 30 параллельно соединенных резисторов с сопротивлениями 150 Ом. Каждый резистор, согласно документации, способен рассеивать до 1 Вт мощности. Используя изученные формулки, мы можем посчитать, что в сумме такая система обладает сопротивлением

  1. и теоретически может рассеивать мощность

Ну, с сопротивлением вопросов нет, оно действительно равно 5 Ом. Ну, плюс-минус 5 % на допуск резисторов, что в данном случае вообще не критично. А вот с мощностью тут не так все однозначно. Помните про закон Джоуля-Ленца, который мы рассматривали? Резисторы будут греться, причем не слабо.

Как показывает практика, если нагружать резисторы по полной, то есть рассеивать на каждом по 1 Вт, то в течении нескольких секунд их температура улетит за 150 градусов. Такая высокая температура критична для резистора и может привести к его разрушению.

Я был готов к такому развитию событий, поэтому заложил для платы нагревателя максимальное напряжение 9 вольт. Это значит, что на каждом резисторе будет выделяться

  • что почти в два раза меньше максимально допустимой мощности в 1 Вт. В сумме на всей плате выделялось, соответственно

Эксперимент показал, что резисторы достигли температуры с комнатных 25 градусов до критичных 120 градусов приблизительно за 10 секунд работы и температура продолжала уверенно расти.

Очевидно, если оставить на длительное время включенным такой нагреватель при комнатной температуре, он неминуемо выйдет из строя.

Возможно, при работе на минус 55 градусах перегрев бы не был столь критичным, однако хотелось исключить вариант спалить плату на столе, поэтому я понизил напряжение, подаваемое на плату на 3 вольта: стал подавать 6 вольт. Теперь на каждом резисторе рассеивалось

  1. а на всей плате

Теперь температура поднималась до 100-110 градусов примерно за 30-40 секунд работы и оставалась на этом уровне (выходила в точку термодинамического равновесия). Эта температура вполне подходит для нагревателя.

Однако пока это были лишь эксперименты на столе при комнатной температуре, главный эксперимент – в термокамере на минус 55 градусах – впереди. Возможно, по его результатам потребуется чуть увеличить рассеиваемую мощность.

А может все останется как есть и этой мощности будет достаточно для вывода девайса на режим за адекватное время, время покажет .

На сегодня все, господа. Удачи вам и до новых встреч!

Источник: http://myelectronix.ru/postoyannyy-tok/40-parallelnoe-soedinenie-rezistorov

Как отличается параллельное и последовательное соединение резисторов?

Большое разнообразие схем основано на двух видах соединений – последовательное параллельное. Для каждого типа существуют свои собственные законы и принципы. Именно это и позволяет создавать устройства с самыми различными техническими параметрами, в том числе и резисторы. Что же такое резистор?

Резистор – радиодеталь, созданная для контроля напряжения и тока в цепи, увеличивая либо понижая его. Резисторы могут быть двух видов – постоянные и переменные. Так, например, светодиоды требуют для себя совсем небольшого тока. Для этого в электрическую цепочку перед светодиодом устанавливается резистор, который обеспечивает необходимое напряжение для работы последнего.

В статье подробны рассмотрены все аспекты последовательного и параллельного подключения резисторов. Бонусом к статье являются видеоролик и детальная информационная статья на рассматриваемую тему.

Последовательное подключение

Начнем с последовательного соединения. По этой схеме каждый резистор подключается с другим только в одной точке, их может быть в цепи 2, 3 и больше. Обозначим сопротивления: R1, R2, R3 и напряжение источника в цепи Uц. При подключении источника питания в ней начнет протекать ток Iц. В цепи с последовательным соединением ток протекает по всем резисторам один за другим.

Поскольку ток течет через все резисторы их сопротивления и ток суммируется, Iц = I1+I2+I3, Rц = R1 +R2 + R3, чем больше отдельно взятое сопротивление, тем тяжелее электронам преодолевать участок цепи. Мощность резисторов при последовательном и параллельном соединении рассчитывается по разным формулам. В последовательных цепях — складываем, в параллельных — это обратно пропорциональная величина.

Последовательное соединение характеризуется тем, что элементы идут друг за другом. Конец одного подключается к началу другого. При подключении полученной цепочки к источнику тока получается кольцо.

Теоретическая часть

Последовательное соединение характерно тем, что через все элементы протекает ток одинаковой силы. То есть, если цепочка состоит из двух резисторов R1 и R2 (как на рисунке ниже), то ток протекающий через каждое из них и любую другую часть цепи будет одинаковой (I = I1 = I2).

Суммарное сопротивление всей цепи последовательно соединенных резисторов считается как сумма сопротивлений всех ее элементов. То есть, номиналы складывают. R = R1 + R2 — это и есть формула расчета сопротивления при последовательном соединении резисторов. Если элементов больше двух, будет просто больше слагаемых.

Еще одно свойство последовательного соединения — на каждом элементе напряжение отличается. Ток в цепи одинаковый, а напряжение на резисторе зависит от его номинала.

Последовательное подключение.

Примеры расчета

Давайте рассмотрим пример. Цепь представлена на рисунке выше. Есть источник тока и два сопротивления. Пусть R1=1,2 кОм, R2= 800 Ом, а ток в цепи 2 А. По закону Ома U = I * R. Подставляем наши значения:

  • U1 = R1 * I = 1200 Ом * 2 А = 2400 В;
  • U2 = R2 * I = 800 Ом * 2А = 1600 В.

Будет интересно➡  SMD резисторы: что это такое и для чего используются?

Общее напряжение цепи считается как сумма напряжений на резисторах: U = U1 + U2 = 2400 В + 1600 В = 4000 В. Полученную цифру можно проверить. Для этого найдем суммарное сопротивление цепи и умножим его на ток.   R = R1 + R2 = 1200 Ом + 800 Ом = 2000 Ом.

Если подставить в формулу напряжения при последовательном соединении сопротивлений, получаем: U = R * I = 2000 Ом * 2  А = 4000 В. Получаем, что общее напряжение данной цепи 4000 В.

А теперь посмотрите на схему. На первом вольтметре (возле резистора R1) показания будут 2400 В, на втором  — 1600 В.  При этом напряжение источника питания — 4000 В. Последовательное соединение – это соединение двух или более резисторов в форме цепи, в которой каждый отдельный резистор соединяется с другим отдельным резистором только в одной точке.

Материал по теме: Как проверить варистор мультиметром.

Общее сопротивление Rобщ

При таком соединении, через все резисторы проходит один и тот же электрический ток. Чем больше элементов на данном участке электрической цепи, тем «труднее» току протекать через него. Следовательно, при последовательном соединении резисторов их общее сопротивление увеличивается, и оно равно сумме всех сопротивлений.

Параллельное соединение резисторов

Параллельное соединение – это соединение, при котором резисторы соединяются между собой обоими контактами. В результате к одной точке (электрическому узлу) может быть присоединено несколько резисторов.

Параллельное подключение резисторов.

Общее сопротивление Rобщ

При таком соединении, через каждый резистор потечет отдельный ток. Сила данного тока будет обратно пропорциональна сопротивлению резистора. В результате общая проводимость такого участка электрической цепи увеличивается, а общее сопротивление в свою очередь уменьшается.

Таким образом, при параллельном подсоединении резисторов с разным сопротивлением, общее сопротивление будет всегда меньше значения самого маленького отдельного резистора.

Если посмотреть на изображение параллельного соединения, заметно, что ко всем элементам прилагается одинаковое напряжение.

То есть, при параллельном соединении резисторов, на каждом из них будет одинаковое напряжение U = U1 = U2 = U3. Получается, что ток разделяется на несколько «ручейков».

То есть, при параллельном соединении резисторов сила тока, протекающего через каждый из элементов, отличается. I = I1+I2+I3. И зависит сила тока (согласно тому же закону Ома) от сопротивления каждого участка цепи.

В случае с параллельным соединением резисторов — от их номинала.

Предлагаем также почитать интересный материал про малоизвестные факты о двигателях постоянного тока в другой нашей статье.

Схема параллельного соединения

Общее сопротивление участка цепи при таком соединении становится ниже. Его высчитывают по формуле: 1/R = 1/R1 + 1/R + 1/R3+. Такая форма хоть и понятна, но неудобна.

Формула расчета сопротивления параллельно подключенных резисторов получается тем сложнее, чем больше элементов соединены параллельно.

Но больше двух-трех редко кто объединяет, так что на практике достаточно знать только две формулы приведенные ниже.

Если подставить значения в эти формулы, то заметим, что результат будет меньше, чем сопротивление резистора с наименьшим номиналом. Это стоит запомнить: результирующее сопротивление включенных параллельно резисторов будет ниже самого маленького номинала. Давайте сначала рассчитаем параллельное соединение двух резисторов разного номинала и посмотрим что получится.

Соединили параллельно 150 Ом и 100 Ом. Считаем результирующее: 150*100 / (150+100) = 15000/250 = 60 Ом. Если соединить 150 Ом и 50 Ом, получим: 150*50 / (150+50) = 7500 / 200 = 37,5 Ом.

 Как видим, в обоих случаях результат оказывается меньше чем самый низкий номинал соединенных деталей. Этим и пользуются, если в наличии нет сопротивления небольшого номинала.

Проблема только в том, что подбирать сложновато: надо каждый раз считать используя калькулятор.

Как высчитывать сопротивление составных резисторов

Возможно, вам будет проще, если знать, что соединив два одинаковых резистора параллельно, получим результат в два раза меньше. Например, соединив параллельно два резистора по 100 Ом получим составное сопротивление 50 Ом. Проверим? Считаем: 100*100 / (100+100) = 10000 / 200 = 50 Ом. При соединении параллельно трех резисторов, считать приходится больше, так как формула сложнее.

Если подключить параллельно 150 Ом, 100 Ом и 50 Ом, результирующее будет 27,3 Ом. Попробуем с более низкими номиналами. Если параллельно включены 20 Ом, 15 Ом и 10 Ом. Получим результирующее сопротивление 4,61 Ом. Вот вам подтверждение правила. Суммарное сопротивление параллельно соединенных резисторов меньше чем самый низкий номинал.

Параллельное соединение резисторов

Параллельное соединение резисторов это соединение, в котором начала всех резисторов соединены в одну общую точку (А), а концы в другую общую точку. При этом по каждому резистору течет свой ток. При параллельном соединении при протекании тока из точки А в точку Б, он имеет несколько путей.

 Таким образом, увеличение числа параллельно соединенных резисторов ведет к увеличению путей протекания тока, то есть к уменьшению противодействия протеканию тока. А это значит, чем большее количество резисторов соединить параллельно, тем меньше станет значение общего сопротивления такого участка цепи.

Общее сопротивление параллельно соединенных резисторов определяется следующим отношением: 1/Rобщ= 1/R1+1/R2+1/R3+…+1/Rn.

Формулы расчета параллельного и последовательного подключения.

Следует отметить, что здесь действует правило «меньше – меньшего». Это означает, что общее сопротивление всегда будет меньше сопротивления любого параллельно включенного резистора. Общее сопротивление для двух параллельно соединенных резисторов рассчитывается по следующей формуле: Rобщ= R1*R2/R1+R2.

Если имеет место два параллельно соединенных резистора с одинаковыми сопротивлениями, то их общее сопротивление будет равно половине сопротивления одного из них. Параллельное соединение резисторов. При параллельном соединении резисторов нескольких приемников они включаются между двумя точками электрической цепи, образуя параллельные ветви.

Заменяя лампы резисторами с сопротивлениями R1, R2, R3, При параллельном соединении ко всем резисторам приложено одинаковое напряжение U. Поэтому согласно закону Ома: I1=U/R1; I2=U/R2; I3=U/R3.

Источник: https://ElectroInfo.net/radiodetali/rezistory/kak-otlichaetsja-parallelnoe-i-posledovatelnoe-soedinenie-rezistorov.html

Расчет сопротивления двух параллельно соединенных резисторов. Последовательное и параллельное соединение резисторов. Формула для расчета параллельного соединения сопротивлений

В каждой электрической схеме присутствует резистор, имеющий сопротивление электрическому току. Резисторы бывают двух типов: постоянные и переменные. Во время разработки любой электрической схемы и ремонта электронных изделий часто приходится применять резистор, обладающий необходимым номиналом.

Несмотря на то что для резисторов предусмотрены различные номиналы , может случиться так, что не будет возможности найти необходимый или же вообще ни один элемент не сможет обеспечить требуемый показатель.

Рассчитать производительность и работу

Угол сдвига фаз вычисляется по изображению указателя. Чтобы иметь возможность определять мощности, поглощаемые схемой, предыдущие формулы используются снова. Для определения работы используются следующие формулы.

Дальнейший интересный контент по теме
Резисторы переменного тока представляют собой омические, индуктивные и емкостные резисторы. Для параллельного подключения таких резисторов в цепи переменного тока применяются разные законы, чем для сопротивлений в цепи постоянного тока. Учитывая это, например, катушку: настоящая катушка имеет как индуктивное, так и омическое сопротивление и поэтому может рассматриваться как последовательная связь чисто индуктивного и чисто омического резистора.

Решением этой проблемы может стать применение последовательного и параллельного соединения. Ознакомившись с этой статьей, вы узнаете об особенностях выполнения расчета и подбора различных номиналов сопротивлений.

Часто при изготовлении какого-либо устройства используют резисторы, которые соединяются в соответствии с последовательной схемой. Эффект от применения такого варианта сборки сводится к увеличению общего сопротивления цепи. Для данного варианта соединения элементов создаваемое ими сопротивление рассчитывается как сумма номиналов. Если же сборка деталей выполняется по параллельной схеме, то здесь потребуется рассчитать сопротивление , используя нижеописанные формулы.

Примеры параллельного соединения проводников

Мы рассматриваем только индуктивную составляющую резистора, т.е. катушку как чисто индуктивный резистор. Аналогично, используется омическое сопротивление и емкостное сопротивление, поскольку омическое сопротивление также может иметь индуктивный компонент. В то время как в случае сопротивления проволоки витки, подобные виткам, видны напрямую, это обычно скрыто в резисторах слоя. Фактически, проводящий слой наносят на носитель, из которого материал, проводящий материал, удаляется с помощью процесса спирально-циркулирующего фрезерования, так что остается спирально циркулирующий слой.

К схеме параллельного соединения прибегают в ситуации, когда стоит задача по снижению суммарного сопротивления, а, помимо этого, увеличения мощности для группы элементов, подключенных по параллельной схеме, которое должно быть больше, чем при их отдельном подключении.

Таким образом генерируется требуемое значение сопротивления. Сразу видно, что эта катушечная структура приводит к индуктивному компоненту. Однако это обычно настолько мало, что его можно пренебречь. Общая обработка взаимосвязи любых резисторов переменного тока невозможна и не требуется с помощью математических знаний, доступных в школе.

Объяснение Подключение серии и параллельное соединение

Ниже приведен упрощенный случай параллельной схемы чисто омического, индуктивного и емкостного резисторов. В этой статье мы рассмотрим параллельное соединение и последовательное соединение резисторов. Давайте сначала уточним, что такое последовательное соединение и что такое параллельное соединение, и где разница между последовательным соединением и параллельным соединением. В последовательной цепи мы имеем два или более сопротивления последовательно. Тот же ток протекает через все резисторы.

Расчет сопротивления

В случае подключения деталей друг с другом, с применением параллельной схемы для расчета суммарного сопротивления, будет использоваться следующая формула:

R(общ)=1/(1/R1+1/R2+1/R3+1/Rn).

  • R1- R3 и Rn – резисторы, подсоединенные по параллельной схеме.

Причем, если цепь создается на основе только двух элементов, то для определения суммарного номинального сопротивления следует использовать такую формулу:

Универсальная схема расчета

На следующем графике показаны резисторы последовательно, два резистора, индивидуально нарисованные в начале, и три резистора под электрической цепью. Напротив, существует параллельное соединение резисторов. Что такое параллельная схема? Теперь, в параллельной цепи, линия распадается, и, следовательно, и ток разлагается. В случае параллельного подключения резисторов во многих случаях впервые рассматривается параллельное соединение двух резисторов. Это выглядит следующим образом, включая формулу для расчета.

R(общ)=R1*R2/R1+R2.

  • R(общ) – суммарное сопротивление;
  • R1 и R2 – резисторы, подсоединенные по параллельной схеме.

Универсальная схема расчета

Применительно к радиотехнике следует уделить внимание одному важному правилу: если подключаемые друг к другу элементы по параллельной схеме имеют одинаковый показатель , то для расчета суммарного номинала необходимо общее значение разделить на число подключенных узлов:

Для трех резисторов в параллельной схеме это будет выглядеть на следующем графике, включая формулу для расчета. Разностное соединение и параллельное соединение. В случае последовательной цепи все резисторы подключаются по одной линии за другой. В случае параллельной схемы, с другой стороны, линия расщепляется, резисторы лежат в отдельных линиях. В последовательной цепи тот же ток протекает через все резисторы, а в случае параллельной цепи ток расщепляется. В случае параллельной схемы одно и то же напряжение подается на каждый резистор, но не в последовательной цепи. Другое примечание: смесь последовательной цепи и параллельной схемы называется групповой схемой.

  • R(общ) – суммарное значение сопротивления;
  • R – номинал резистора, подсоединенного по параллельной схеме;
  • n – число подключенных узлов.

Особое внимание следует обратить на то, что конечный показатель сопротивления в случае использования параллельной схемы подключения обязательно будет меньше по сравнению с номиналом любого элемента, подключаемого в цепь.

Примеры Подключение серий и параллельное соединение

В следующих примерах мы увидим, как рассчитать смесь схемы параллельной цепи и серии. В этой области шаг за шагом должен быть рассчитан набор последовательных схем и параллельных схем. На следующем графике показана смесь последовательного соединения и параллельного соединения. Каково общее сопротивление?

Сначала мы суммируем 20 Ом и 30 Ом, так как здесь имеется параллельная схема. Таким образом, схема выглядит следующим образом. Теперь добавим эту схему, добавив резисторы для вычисления общего сопротивления. Вычислите общее сопротивление следующего контура.

Пример расчета

Для большей наглядности можно рассмотреть следующий пример: допустим, у нас есть три резистора, чьи номиналы соответственно равны 100, 150 и 30 Ом. Если воспользоваться первой формулой для определения общего номинала, то получим следующее:

Прежде всего, вы должны увидеть, что есть короткое замыкание на резисторе с 95 Ом. Поэтому ток течет практически полностью по линии ниже, а 95 Ом не учитывается при расчете полного сопротивления. В противном случае у нас есть сочетание последовательной цепи и параллельной схемы.

Комбинированные последовательные и параллельные схемы

Вам нужно знать, как рассчитать резисторы последовательно, параллельно и комбинацию резисторов параллельно и последовательно? Если вы не хотите жарить свою печатную плату, вы это делаете! Эта статья покажет вам, как это сделать за несколько простых шагов. Это просто образный способ говорить, чтобы понятий было легко понять.

Некоторые факты, которые вы должны учитывать
Любой материал, который проводит электрический ток, имеет удельное сопротивление, которое представляет собой сопротивление материала при прохождении электрического тока.
  • Понять понятие сопротивления.
  • Единицей измерения резисторов является Ом.
Параллельное соединение резисторов характеризуется тем, что входные клеммы каждого из резисторов соединены друг с другом.

R(общ)=1/(1/100+1/150+1/30)=1/(0,01+0,007+0,03)=1/0,047=21,28Ом.

Если выполнить несложные расчеты, то можно получить следующее: для цепи, включающей в себя три детали, где наименьший показатель сопротивления составляет 30 Ом, результирующее значение номинала будет равно 21,28 Ом. Этот показатель будет меньше минимального значения номинала в цепи практически на 30%.

Аналогично, в конфигурации параллельного резистора выходные клеммы также соединены друг с другом. Из-за этого все резисторы пропускают одинаковое напряжение, т.е. имеют одинаковое падение напряжения. Это связано с тем, что концы каждого из резисторов соединены с одной и той же точкой в ​​цепи, и поэтому они имеют одинаковое напряжение.

Однако общий ток, протекающий через резисторы параллельно, равен сумме интенсивностей, которые проходят каждый резистор. Дифференциация параллельного соединения из последовательного интерфейса проста. В последовательной конфигурации резистора выходной разъем одного подключается к входному разъему следующего.

Важные нюансы

Обычно для резисторов параллельное соединение применяется тогда, когда стоит задача по созданию сопротивления большей мощности. Для ее решения потребуются резисторы, которые должны иметь равные показатели сопротивления и мощности. При таком варианте определить общую мощность можно следующим образом : мощность одного элемента необходимо перемножить с суммарным числом всех резисторов, из которых состоит цепь, подсоединенных друг с другом в соответствии с параллельной схемой.

Вычисление сопротивлений параллельно: формула

Чтобы вычислить эквивалентное сопротивление нескольких подключенных параллельных резисторов, мы должны применить формулу, указанную выше этих строк. Чтобы избежать ошибок в расчетах, лучше всего разделить формулу на два шага. Сначала мы вычисляем сумму обратного для каждого сопротивления и, когда получаем результат, вычисляем его обратно, чтобы знать эквивалентное сопротивление.

Решенное сопротивление сопротивлениям параллельно

Например, мы вычислим эквивалентное сопротивление конфигурации, аналогичное той, что мы имеем на следующем рисунке. Первый шаг: вычислить сумму обратного каждого сопротивления. Шаг второй: вычислите обратное только что полученное сопротивление.

Вычисление трех резисторов параллельно
Если мы хотим решить предыдущий пример, но используя наш калькулятор из трех резисторов параллельно в сети, просто заполните значение каждого резистора в соответствующем поле. Порядок, в котором вы его пишете, не имеет значения, поэтому вам не нужно его уважать.

Скажем, если нами будут использоваться пять резисторов, чей номинал составляет 100 Ом, а мощность каждого равна 1 Вт, которые присоединены друг к другу в соответствии с параллельной схемой, то суммарный показатель сопротивления будет равен 20 Ом, а мощность составит 5 Вт.

Если взять те же резисторы, но подсоединить их в соответствии с последовательной схемой, то конечная мощность составит 5 Вт, а суммарный номинал будет равен 500 Ом.

Когда вы пишете значение трех резисторов параллельно, просто нажмите кнопку расчета, и вы автоматически получите результат без применения формулы для расчета сопротивления параллельно. С этим вы экономите время и, прежде всего, просчеты. Как мы видели в предыдущих разделах, устройства, которые выступают против прохода электрического тока более выраженным образом, чем обычно, обычно используются в электрических цепях. Эти устройства называются резисторами и могут быть связаны таким образом, что вместе они эквивалентны значению другого сопротивления, называемого эквивалентным сопротивлением.

Заключение

Параллельная схема подключения резисторов очень востребована по той причине, что часто возникает задача по созданию такого номинала, которого невозможно добиться при помощи простого параллельного соединения. При этом процедура расчета этого параметра отличается достаточной сложностью , где необходимо учитывать разные параметры.

Ассоциация резисторов в серии

Он называется полученным сопротивлением или эквивалентом, к значению сопротивления, которое получается путем связывания их набора. В основном резисторы могут быть связаны последовательно, параллельно или комбинацией обоих смешанных вызовов. Когда два или более резисторов последовательно, интенсивность тока, проходящая через каждую из них, одинакова.

Если применить закон Ома к каждому из сопротивлений предыдущего рисунка, мы получим. Если мы сделаем сумму от члена к элементу по трем уравнениям, заметим, что. Таким образом, приведенное выше уравнение, если учесть, что. Итак, если вы понимаете, вы можете видеть, что три предыдущих резистора серии эквивалентны одному резистору, значение которого представляет собой сумму трех предыдущих.

Здесь важная роль отводится не только количеству подключаемых элементов, но и рабочим параметрам резисторов — прежде всего, сопротивлению и мощности. Если один из подключаемых элементов будет иметь неподходящий показатель, то это не позволит эффективно решить задачу по созданию требуемого номинала в цепи.

Последовательное соединение это соединение двух или более резисторов в форме цепи, в которой каждый отдельный резистор соединяется с другим отдельным резистором только в одной точке.

Ассоциация резисторов в параллельном

Когда два или более резисторов параллельны, они делятся своими концами, как показано на следующем рисунке. Если мы применим закон Ома в каждом из сопротивлений фигуры. Зная, что сумма интенсивностей каждого сопротивления равна интенсивности перед входом и выходом из набора, образованного тремя сопротивлениями.

Ассоциация смешанного сопротивления

Как правило, в электрических цепях они не просто похожи на последовательные или параллельные резисторы, но и на комбинацию обоих. Чтобы лучше понять, как подойти к этим типам ассоциаций, мы проиллюстрируем пример. Представьте себе следующую схему сопротивлений.

Общее сопротивление R общ

При таком соединении, через все резисторы проходит один и тот же электрический ток. Чем больше элементов на данном участке электрической цепи, тем «труднее» току протекать через него. Следовательно, при последовательном соединении резисторов их общее сопротивление увеличивается, и оно равно сумме всех сопротивлений.

Подключение 2 равных громкоговорителей последовательно добавляет импедансы и ватты? Сопротивление добавляется, и общая мощность рассеивания на громкоговоритель уменьшается вдвое. Предположим, что выход 8 вольт и динамик 8 Ом ток, который циркулирует громкоговорителем.

Таким образом, динамик должен будет поддерживать более 8 Вт, чтобы он не был поврежден. Теперь подключите два динамика 8 Ом и 8 Вт последовательно, импеданс обоих составляет 16 Ом. Теперь мы вычисляем ток, протекающий через динамики. С этими данными мы вычисляем мощность в каждом динамике.

Напряжение при последовательном соединении

Напряжение при последовательном соединении распределяется на каждый резистор согласно закону Ома:

Т.е чем большее сопротивление резистора, тем большее напряжение на него падает.

Вывод: динамики работают более сдержанно, но усилитель будет поставлять половину мощности, когда динамик составляет 8 Ом. Разделен ли импеданс и ватт параллельно? Если мы рассмотрим одни и те же ораторы на примере предыдущего вопроса, можно сказать, что общий импеданс уменьшается наполовину и рассчитывается следующим образом.

Для работы с более чем двумя динамиками параллельно необходимо использовать другое уравнение для расчета импеданса. Если мы анализируем отдельно каждого динамика, мы понимаем, что каждый из них ведет себя так же, как в примере 1, когда каждый из них подключен к усилителю, через который они будут циркулировать 1 А, а мощность, подлежащая рассеиванию, будет составлять 8 Вт в каждом динамике.

Параллельное соединение это соединение, при котором резисторы соединяются между собой обоими контактами. В результате к одной точке (электрическому узлу) может быть присоединено несколько резисторов.

Общее сопротивление R общ

При таком соединении, через каждый резистор потечет отдельный ток. Сила данного тока будет обратно пропорциональна сопротивлению резистора. В результате общая проводимость такого участка электрической цепи увеличивается, а общее сопротивление в свою очередь уменьшается.

Таким образом, при параллельном подсоединении резисторов с разным сопротивлением, общее сопротивление будет всегда меньше значения самого маленького отдельного резистора.

Формула общей проводимости при параллельном соединении резисторов:

Формула эквивалентного общего сопротивления при параллельном соединении резисторов:

Для двух одинаковых резисторов общее сопротивление будет равно половине одного отдельного резистора:

Соответственно, для n одинаковых резисторов общее сопротивление будет равно значению одного резистора, разделенного на n.

Напряжение при параллельном соединении

Напряжение между точками A и B является как общим напряжением для всего участка цепи, так и напряжением, падающим на каждый резистор в отдельности. Поэтому при параллельном соединении на все резисторы упадет одинаковое напряжение.

Через каждый резистор течет ток, сила которого обратно пропорциональна сопротивлению резистора. Для того чтобы узнать какой ток течет через определенный резистор, можно воспользоваться законом Ома:

Смешанным соединением называют участок цепи, где часть резисторов соединяются между собой последовательно, а часть параллельно. В свою очередь, смешанное соединение бывает последовательного и параллельного типов.

Общее сопротивление R общ

  • Цепь разбивают на участки с только пареллельным или только последовательным соединением.
  • Вычисляют общее сопротивление для каждого отдельного участка.
  • Вычисляют общее сопротивление для всей цепи смешанного соединения.

Также существует более быстрый способ расчета общего сопротивления для смешанного соединения. Можно, в соответствии схеме, сразу записывать формулу следующим образом:

  • Если резисторы соединяются последоватеьно — складывать.
  • Если резисторы соединяются параллельно — использовать условное обозначение «||».
  • Подставлять формулу для параллельного соединения где стоит символ «||».

Так это будет выглядеть для схемы 1:

Сопротивление

параллельно — параллельно подключенные резисторы

В отличие от предыдущей схемы последовательного резистора, в параллельной резисторной сети ток схемы может проходить по нескольким путям, поскольку существует несколько путей для тока. Тогда параллельные цепи классифицируются как делители тока.

Поскольку существует несколько путей прохождения тока питания, ток может быть неодинаковым во всех ответвлениях параллельной сети. Однако падение напряжения на всех резисторах в параллельной резистивной сети одинаково.Тогда резисторы в параллельном соединении имеют общее напряжение на них, и это верно для всех параллельно соединенных элементов.

Таким образом, мы можем определить параллельную резистивную цепь как такую, в которой резисторы подключены к одним и тем же двум точкам (или узлам), и идентифицируется по тому факту, что у нее более одного пути тока, подключенного к общему источнику напряжения. Затем в нашем примере с параллельным резистором ниже напряжение на резисторе R 1 равно напряжению на резисторе R 2 , которое равно напряжению на R 3 и которое равно напряжению питания.Следовательно, для параллельной сети резисторов это определяется как:

В следующих резисторах, включенных в параллельную цепь, резисторы R 1 , R 2 и R 3 соединены вместе параллельно между двумя точками A и B, как показано.

Цепь параллельного резистора

В предыдущей схеме последовательных резисторов мы видели, что полное сопротивление цепи R T равно сумме всех отдельных резисторов, сложенных вместе.Для резисторов, включенных параллельно, сопротивление схемы замещения R T рассчитывается иначе.

Здесь обратные (1 / R) значения отдельных сопротивлений складываются вместе вместо самих сопротивлений с обратной алгебраической суммой, дающей эквивалентное сопротивление, как показано.

Уравнение параллельного резистора

Тогда величина, обратная эквивалентному сопротивлению двух или более резисторов, соединенных параллельно, является алгебраической суммой обратных величин отдельных сопротивлений.

Если два параллельных сопротивления или импеданса равны и имеют одинаковое значение, то полное или эквивалентное сопротивление R T равно половине значения одного резистора. Это равно R / 2 и для трех одинаковых резисторов, включенных параллельно, R / 3 и т. Д.

Обратите внимание, что эквивалентное сопротивление всегда меньше, чем наименьшее сопротивление в параллельной сети, поэтому общее сопротивление R T всегда будет уменьшаться по мере добавления дополнительных параллельных резисторов.

Параллельное сопротивление дает нам значение, известное как Проводимость , символ G с единицами проводимости Siemens , символ S .Электропроводность является обратной или обратной величине сопротивления (G = 1 / R). Чтобы преобразовать проводимость обратно в значение сопротивления, нам нужно взять обратную проводимость, что даст нам общее сопротивление, R T резисторов, включенных параллельно.

Теперь мы знаем, что резисторы, подключенные между одними и теми же двумя точками, считаются параллельными. Но параллельная резистивная цепь может принимать множество форм, отличных от очевидной, приведенной выше, и вот несколько примеров того, как резисторы могут быть соединены вместе параллельно.

Различные схемы параллельных резисторов

Пять вышеупомянутых резистивных цепей могут отличаться друг от друга, но все они организованы как резисторов в параллельном соединении , и поэтому применяются одни и те же условия и уравнения.

Резисторы в параллели Пример №1

Найдите полное сопротивление R T следующих резисторов, включенных в параллельную сеть.

Общее сопротивление R T на двух клеммах A и B рассчитывается как:

Этот метод взаимного расчета может использоваться для расчета любого количества отдельных сопротивлений, соединенных вместе в одной параллельной сети.

Если, однако, есть только два отдельных резистора, подключенных параллельно, мы можем использовать гораздо более простую и быструю формулу, чтобы найти полное или эквивалентное значение сопротивления, R T , и немного сократить взаимные математические вычисления.

Этот гораздо более быстрый метод вычисления двух параллельно включенных резисторов, имеющих равные или неодинаковые значения, выражается как:

Резисторы в параллели Пример №2

Рассмотрим следующую схему, в которой только два резистора включены в параллельную комбинацию.

Используя приведенную выше формулу для двух параллельно соединенных резисторов, мы можем рассчитать полное сопротивление цепи R T как:

Один важный момент, который следует помнить о резисторах, включенных параллельно, заключается в том, что полное сопротивление цепи (R T ) любых двух резисторов, соединенных вместе параллельно, всегда будет на МЕНЬШЕ , чем значение наименьшего резистора в этой комбинации.

В нашем примере выше значение комбинации было рассчитано как: R T = 15 кОм, где значение наименьшего резистора составляет 22 кОм, что намного больше.Другими словами, эквивалентное сопротивление параллельной сети всегда будет меньше, чем у наименьшего отдельного резистора в комбинации.

Кроме того, в случае, если 1 рэндов равно значению 2 рэнд, то есть 1 рэнд = 2 рэнд, общее сопротивление сети будет ровно половиной значения одного из резисторы, R / 2.

Аналогично, если три или более резистора, каждый с одинаковым номиналом, подключены параллельно, то эквивалентное сопротивление будет равно R / n, где R — номинал резистора, а n — количество отдельных сопротивлений в комбинации.

Например, шесть резисторов 100 Ом соединены параллельно. Таким образом, эквивалентное сопротивление будет: R T = R / n = 100/6 = 16,7 Ом. Но учтите, что это работает ТОЛЬКО для эквивалентных резисторов. То есть все резисторы имеют одинаковое значение.

Токи в параллельной цепи резистора

Полный ток I T , входящий в параллельную резистивную цепь, представляет собой сумму всех отдельных токов, протекающих во всех параллельных ветвях.Но величина тока, протекающего через каждую параллельную ветвь, не обязательно может быть одинаковой, поскольку значение сопротивления каждой ветви определяет количество тока, протекающего в этой ветви.

Например, хотя параллельная комбинация имеет одинаковое напряжение на ней, сопротивления могут быть разными, поэтому ток, протекающий через каждый резистор, определенно будет отличаться, как определено законом Ома.

Рассмотрим два резистора, включенных параллельно выше. Ток, протекающий через каждый из резисторов (I R1 и I R2 ), соединенных параллельно, не обязательно имеет одинаковое значение, поскольку он зависит от значения сопротивления резистора.Однако мы знаем, что ток, который входит в цепь в точке A, должен также выходить из цепи в точке B.

Закон Кирхгофа по току гласит: « полный ток, выходящий из цепи, равен току, входящему в цепь — ток не теряется ». Таким образом, полный ток, протекающий в цепи, определяется как:

I T = I R1 + I R2

Используя закон Ома , мы можем вычислить ток, протекающий через каждый параллельный резистор, показанный в Примере №2 выше, как:

Ток, протекающий в резисторе R 1 , определяется как:

I R1 = В S ÷ R 1 = 12 В ÷ 22 кОм = 0.545 мА или 545 мкА

Ток, протекающий через резистор R 2 , определяется как:

I R2 = В S ÷ R 2 = 12 В ÷ 47 кОм = 0,255 мА или 255 мкА

, что дает нам полный ток I T , протекающий по цепи как:

I T = 0,545 мА + 0,255 мА = 0,8 мА или 800 мкА

, и это также можно проверить напрямую с помощью закона Ома:

I T = V S ÷ R T = 12 ÷ 15 кОм = 0.8мА или 800мкА (то же)

Уравнение для расчета полного тока, протекающего в параллельной цепи резистора, который представляет собой сумму всех отдельных токов, сложенных вместе, дается как:

I всего = I 1 + I 2 + I 3 … .. + I n

Тогда параллельные резистивные цепи можно также рассматривать как «делители тока», потому что ток питания разделяется или делится между различными параллельными ветвями. Таким образом, схема параллельного резистора, имеющая резистивные цепи N , будет иметь N разных путей тока, сохраняя при этом общее напряжение на себе.Параллельные резисторы также можно менять местами без изменения общего сопротивления или общего тока цепи.

Резисторы в параллели Пример №3

Рассчитайте отдельные токи ответвления и общий ток, потребляемый от источника питания для следующего набора резисторов, соединенных вместе в параллельной комбинации.

Поскольку напряжение питания является общим для всех резисторов в параллельной цепи, мы можем использовать закон Ома для расчета тока отдельной ветви следующим образом.

Тогда полный ток цепи I T , протекающий в параллельную комбинацию резисторов, будет:

Это полное значение тока цепи в 5 ампер также можно найти и проверить, найдя эквивалентное сопротивление цепи R T параллельной ветви и разделив его на напряжение питания V S следующим образом.

Сопротивление эквивалентной цепи:

Тогда ток в цепи будет:

Резисторы в параллельном соединении

Итак, подведем итоги.Когда два или более резистора соединены так, что оба их вывода соответственно подключены к каждому выводу другого резистора или резисторов, они говорят, что они соединены вместе параллельно. Напряжение на каждом резисторе в параллельной комбинации точно такое же, но токи, протекающие через них, не такие же, поскольку это определяется их значением сопротивления и законом Ома. Тогда параллельные цепи являются делителями тока.

Эквивалентное или полное сопротивление R T параллельной комбинации определяется путем взаимного сложения, и общее значение сопротивления всегда будет меньше, чем у наименьшего отдельного резистора в комбинации.Параллельные цепи резисторов можно менять местами в одной и той же комбинации без изменения общего сопротивления или общего тока цепи. Резисторы, соединенные вместе в параллельную цепь, будут продолжать работать, даже если один резистор может быть разомкнут.

До сих пор мы видели цепи резисторов, соединенные последовательно или параллельно. В следующем уроке о резисторах мы рассмотрим соединение резисторов вместе как в последовательной, так и в параллельной комбинации, в то же время создавая смешанную или комбинационную схему резисторов.

резисторов последовательно и параллельно

Цели обучения

К концу этого раздела вы сможете:

  • Нарисуйте цепь с резисторами, включенными параллельно и последовательно.
  • Рассчитайте падение напряжения тока на резисторе, используя закон Ома.
  • Контраст — способ расчета общего сопротивления для резисторов, включенных последовательно и параллельно.
  • Объясните, почему полное сопротивление параллельной цепи меньше наименьшего сопротивления любого из резисторов в этой цепи.
  • Рассчитайте общее сопротивление цепи, которая содержит смесь резисторов, включенных последовательно и параллельно.

Большинство схем имеет более одного компонента, называемого резистором , который ограничивает поток заряда в цепи. Мера этого предела расхода заряда называется сопротивлением . Простейшие комбинации резисторов — это последовательное и параллельное соединение, показанное на Рисунке 1.Общее сопротивление комбинации резисторов зависит как от их индивидуальных значений, так и от способа их подключения.

Рис. 1. (a) Последовательное соединение резисторов. (б) Параллельное соединение резисторов.

Когда резисторы в серии ? Резисторы включены последовательно всякий раз, когда поток заряда, называемый током , должен проходить через устройства последовательно. Например, если ток течет через человека, держащего отвертку, в землю, тогда R 1 на Рисунке 1 (а) может быть сопротивлением вала отвертки, R 2 сопротивлением ее ручки , R 3 сопротивление тела человека и R 4 сопротивление его обуви.На рисунке 2 показаны резисторы, последовательно подключенные к источнику напряжения . Кажется разумным, что полное сопротивление является суммой отдельных сопротивлений, учитывая, что ток должен проходить через каждый резистор последовательно. (Этот факт был бы преимуществом для человека, желающего избежать поражения электрическим током, который мог бы уменьшить ток, надев обувь с резиновыми подошвами с высоким сопротивлением. прибор, уменьшающий рабочий ток.)

Рис. 2. Три резистора, подключенных последовательно к батарее (слева), и эквивалентное одиночное или последовательное сопротивление (справа).

Чтобы убедиться, что последовательно соединенные сопротивления действительно складываются, давайте рассмотрим потерю электроэнергии, называемую падением напряжения , в каждом резисторе на Рисунке 2. Согласно закону Ома , падение напряжения В на резистор, когда через него протекает ток, рассчитывается по формуле V = IR , где I равно току в амперах (A), а R — сопротивление в омах (Ω).Другой способ представить это: В, — это напряжение, необходимое для протекания тока I через сопротивление R . Таким образом, падение напряжения на R 1 составляет В 1 = IR 1 , что на R 2 составляет В 2 = IR 2 и что через R 3 составляет V 3 = IR 3 .Сумма этих напряжений равна выходному напряжению источника; то есть

V = V 1 + V 2 + V 3 .

Это уравнение основано на сохранении энергии и сохранении заряда. Электрическая потенциальная энергия может быть описана уравнением PE = qV , где q — электрический заряд, а V — напряжение. Таким образом, энергия, подаваемая источником, составляет кв.в. , а энергия, рассеиваемая резисторами, составляет

.

qV 1 + qV 2 + qV 3 .

Установление связей: законы о сохранении

Вывод выражений для последовательного и параллельного сопротивления основан на законах сохранения энергии и сохранения заряда, которые утверждают, что общий заряд и полная энергия постоянны в любом процессе. Эти два закона непосредственно участвуют во всех электрических явлениях и будут многократно использоваться для объяснения как конкретных эффектов, так и общего поведения электричества.

Эти энергии должны быть равны, потому что в цепи нет другого источника и другого назначения для энергии.Таким образом, qV = qV 1 + qV 2 + qV 3 . Плата q отменяется, давая V = V 1 + V 2 + V 3 , как указано. (Обратите внимание, что одинаковое количество заряда проходит через батарею и каждый резистор за заданный промежуток времени, поскольку нет емкости для хранения заряда, нет места для утечки заряда и заряд сохраняется.) Теперь подстановка значений для отдельных напряжений дает

V = IR 1 + IR 2 + IR 3 = I ( R 1 + R 2 + R 3 ).

Обратите внимание, что для эквивалентного сопротивления одной серии R с , мы имеем

В = ИК с .

Это означает, что полное или эквивалентное последовательное сопротивление R с трех резисторов составляет R с = R 1 + R 2 + R 3 .Эта логика действительна в общем для любого количества резисторов, включенных последовательно; таким образом, полное сопротивление R с последовательного соединения составляет

R с = R 1 + R 2 + R 3 +…,

, как предлагается. Поскольку весь ток должен проходить через каждый резистор, он испытывает сопротивление каждого, а последовательно соединенные сопротивления просто складываются.

Пример 1. Расчет сопротивления, тока, падения напряжения и рассеиваемой мощности: анализ последовательной цепи

Предположим, что выходное напряжение батареи на рисунке 2 равно 12.0 В, а сопротивления равны R 1 = 1,00 Ом, R 2 = 6,00 Ом и R 3 = 13,0 Ом. а) Каково полное сопротивление? (б) Найдите ток. (c) Рассчитайте падение напряжения на каждом резисторе и покажите, как они складываются, чтобы равняться выходному напряжению источника. (d) Рассчитайте мощность, рассеиваемую каждым резистором. (e) Найдите выходную мощность источника и покажите, что она равна общей мощности, рассеиваемой резисторами.

Стратегия и решение для (а)

Общее сопротивление — это просто сумма отдельных сопротивлений, определяемая следующим уравнением:

[латекс] \ begin {array} {lll} {R} _ {\ text {s}} & = & {R} _ {1} + {R} _ {2} + {R} _ {3} \ \ & = & 1.00 \ text {} \ Omega + 6.00 \ text {} \ Omega + 13.0 \ text {} \ Omega \\ & = & 20.0 \ text {} \ Omega \ end {array} \\ [/ latex].

Стратегия и решение для (b)

Ток определяется по закону Ома, В = IR . Ввод значения приложенного напряжения и общего сопротивления дает ток для цепи:

[латекс] I = \ frac {V} {{R} _ {\ text {s}}} = \ frac {12.0 \ text {V}} {20.0 \ text {} \ Omega} = 0.60 \ text {A }\\[/латекс].

Стратегия и решение для (c)

Напряжение — или падение IR — на резисторе определяется законом Ома.Ввод значения тока и значения первого сопротивления дает

.

В 1 = IR 1 = (0,600 A) (1,0 Ом) = 0,600 В.

Аналогично

В 2 = IR 2 = (0,600 A) (6,0 Ом) = 3,60 В

и

V3 = IR 3 = (0,600 A) (13,0 Ом) = 7,80 В.

Обсуждение для (c)

Три капли IR добавляют к 12.0 В, прогноз:

V 1 + V 2 + V 3 = (0,600 + 3,60 + 7,80) V = 12,0 В.

Стратегия и решение для (d)

Самый простой способ рассчитать мощность в ваттах (Вт), рассеиваемую резистором в цепи постоянного тока, — это использовать закон Джоуля , P = IV , где P — электрическая мощность. В этом случае через каждый резистор протекает одинаковый полный ток.Подставляя закон Ома V = IR в закон Джоуля, мы получаем мощность, рассеиваемую первым резистором, как

P 1 = I 2 R 1 = (0,600 A) 2 (1,00 Ом) = 0,360 Вт

Аналогично

P 2 = I 2 R 2 = (0,600 A) 2 (6,00 Ом) = 2,16 Вт

и

P 3 = I 2 R 3 = (0.{2}} {R} \\ [/ latex], где В, — падение напряжения на резисторе (а не полное напряжение источника). Будут получены те же значения.

Стратегия и решение для (e)

Самый простой способ рассчитать выходную мощность источника — использовать P = IV , где В, — напряжение источника. Это дает

P = (0,600 A) (12,0 В) = 7,20 Вт.

Обсуждение для (e)

Обратите внимание, что по совпадению общая мощность, рассеиваемая резисторами, также равна 7.20 Вт, столько же, сколько мощность, выдаваемая источником. То есть

P 1 + P 2 + P 3 = (0,360 + 2,16 + 4,68) W = 7,20 Вт.

Мощность — это энергия в единицу времени (ватт), поэтому для сохранения энергии требуется, чтобы выходная мощность источника была равна общей мощности, рассеиваемой резисторами.

Основные характеристики резисторов серии

  1. Последовательные сопротивления добавить: R с = R 1 + R 2 + R 3 +….
  2. Одинаковый ток протекает последовательно через каждый резистор.
  3. Отдельные последовательно включенные резисторы не получают полное напряжение источника, а делят его.

На рисунке 3 показаны резисторы параллельно , подключенные к источнику напряжения. Резисторы включены параллельно, когда каждый резистор подключен непосредственно к источнику напряжения с помощью соединительных проводов с незначительным сопротивлением. Таким образом, к каждому резистору приложено полное напряжение источника. Каждый резистор потребляет такой же ток, как если бы он один был подключен к источнику напряжения (при условии, что источник напряжения не перегружен).Например, автомобильные фары, радио и т. Д. Подключены параллельно, так что они используют полное напряжение источника и могут работать полностью независимо. То же самое и в вашем доме, или в любом другом здании. (См. Рисунок 3 (b).)

Рис. 3. (a) Три резистора, подключенных параллельно батарее, и эквивалентное одиночное или параллельное сопротивление. (б) Электроснабжение в доме. (Источник: Dmitry G, Wikimedia Commons)

Чтобы найти выражение для эквивалентного параллельного сопротивления R p , давайте рассмотрим протекающие токи и их связь с сопротивлением.Поскольку каждый резистор в цепи имеет полное напряжение, токи, протекающие через отдельные резисторы, равны [латекс] {I} _ {1} = \ frac {V} {{R} _ {1}} \\ [/ latex] , [латекс] {I} _ {2} = \ frac {V} {{R} _ {2}} \\ [/ latex] и [латекс] {I} _ {3} = \ frac {V} {{R} _ {3}} \\ [/ латекс]. Сохранение заряда подразумевает, что полный ток I , производимый источником, является суммой этих токов:

I = I 1 + I 2 + I 3 .

Подстановка выражений для отдельных токов дает

[латекс] I = \ frac {V} {{R} _ {1}} + \ frac {V} {{R} _ {2}} + \ frac {V} {{R} _ {3}} = V \ left (\ frac {1} {{R} _ {1}} + \ frac {1} {{R} _ {2}} + \ frac {1} {{R} _ {3}} \ справа) \\ [/ латекс].

Обратите внимание, что закон Ома для эквивалентного одиночного сопротивления дает

[латекс] I = \ frac {V} {{R} _ {p}} = V \ left (\ frac {1} {{R} _ {p}} \ right) \\ [/ latex].

Члены в скобках в последних двух уравнениях должны быть равны. Обобщая для любого количества резисторов, общее сопротивление R p параллельного соединения связано с отдельными сопротивлениями на

.

[латекс] \ frac {1} {{R} _ {p}} = \ frac {1} {{R} _ {1}} + \ frac {1} {{R} _ {2}} + \ гидроразрыв {1} {{R} _ {\ text {.} 3}} + \ text {.} \ Text {…} \\ [/ latex]

Это соотношение приводит к общему сопротивлению R p , которое меньше наименьшего из отдельных сопротивлений. (Это видно в следующем примере.) При параллельном подключении резисторов от источника течет больше тока, чем протекает по любому из них по отдельности, поэтому общее сопротивление ниже.

Пример 2. Расчет сопротивления, тока, рассеиваемой мощности и выходной мощности: анализ параллельной цепи

Пусть выходное напряжение батареи и сопротивления в параллельном соединении на Рисунке 3 будут такими же, как и в ранее рассмотренном последовательном соединении: В = 12.0 В, R 1 = 1,00 Ом, R 2 = 6,00 Ом и R 3 = 13,0 Ом. а) Каково полное сопротивление? (б) Найдите полный ток. (c) Рассчитайте токи в каждом резисторе и покажите, как они складываются, чтобы равняться общему выходному току источника. (d) Рассчитайте мощность, рассеиваемую каждым резистором. (e) Найдите выходную мощность источника и покажите, что она равна общей мощности, рассеиваемой резисторами.

Стратегия и решение для (а)

Общее сопротивление для параллельной комбинации резисторов находится с помощью следующего уравнения.Ввод известных значений дает

[латекс] \ frac {1} {{R} _ {p}} = \ frac {1} {{R} _ {1}} + \ frac {1} {{R} _ {2}} + \ frac {1} {{R} _ {3}} = \ frac {1} {1 \ text {.} \ text {00} \ text {} \ Omega} + \ frac {1} {6 \ text {. } \ text {00} \ text {} \ Omega} + \ frac {1} {\ text {13} \ text {.} 0 \ text {} \ Omega} \\ [/ latex].

Таким образом,

[латекс] \ frac {1} {{R} _ {p}} = \ frac {1.00} {\ text {} \ Omega} + \ frac {0 \ text {.} \ Text {1667}} {\ текст {} \ Omega} + \ frac {0 \ text {.} \ text {07692}} {\ text {} \ Omega} = \ frac {1 \ text {.} \ text {2436}} {\ text { } \ Omega} \\ [/ латекс].

(Обратите внимание, что в этих расчетах каждый промежуточный ответ отображается с дополнительной цифрой.) Мы должны перевернуть это, чтобы найти полное сопротивление R p . Это дает

[латекс] {R} _ {\ text {p}} = \ frac {1} {1 \ text {.} \ Text {2436}} \ text {} \ Omega = 0 \ text {.} \ Text { 8041} \ text {} \ Omega \\ [/ latex].

Общее сопротивление с правильным количеством значащих цифр составляет R p = 0,804 Ом

Обсуждение для (а)

R p , как и предполагалось, меньше наименьшего индивидуального сопротивления.

Стратегия и решение для (b)

Полный ток можно найти из закона Ома, заменив полное сопротивление R p . Это дает

[латекс] I = \ frac {V} {{R} _ {\ text {p}}} = \ frac {\ text {12.0 V}} {0.8041 \ text {} \ Omega} = \ text {14} \ text {.} \ text {92 A} \\ [/ latex].

Обсуждение для (б)

Ток I для каждого устройства намного больше, чем для тех же устройств, подключенных последовательно (см. Предыдущий пример).Схема с параллельным соединением имеет меньшее общее сопротивление, чем резисторы, включенные последовательно.

Стратегия и решение для (c)

Отдельные токи легко вычислить по закону Ома, поскольку каждый резистор получает полное напряжение. Таким образом,

[латекс] {I} _ {1} = \ frac {V} {{R} _ {1}} = \ frac {12.0 \ text {V}} {1.00 \ text {} \ Omega} = 12.0 \ text {A} \\ [/ латекс].

Аналогично

[латекс] {I} _ {2} = \ frac {V} {{R} _ {2}} = \ frac {12.0 \ text {V}} {6.00 \ text {} \ Omega} = 2 \ text {.} \ text {00} \ text {A} \\ [/ latex]

и

[латекс] {I} _ {3} = \ frac {V} {{R} _ {3}} = \ frac {\ text {12} \ text {.} 0 \ text {V}} {\ text {13} \ text {.} \ Text {0} \ text {} \ Omega} = 0 \ text {.} \ Text {92} \ text {A} \\ [/ latex].

Обсуждение для (c)

Общий ток складывается из отдельных токов:

I 1 + I 2 + I 3 = 14,92 A.

Это соответствует сохранению заряда.{2}} {13.0 \ text {} \ Omega} = 11.1 \ text {W} \\ [/ latex].

Обсуждение для (d)

Мощность, рассеиваемая каждым резистором параллельно, значительно выше, чем при последовательном подключении к тому же источнику напряжения.

Стратегия и решение для (e)

Общую мощность также можно рассчитать несколькими способами. Выбирая P = IV и вводя полный ток, получаем

P = IV = (14,92 A) (12,0 В) = 179 Вт.

Обсуждение для (e)

Суммарная мощность, рассеиваемая резисторами, также 179 Вт:

P 1 + P 2 + P 3 = 144 Вт + 24,0 Вт + 11,1 Вт = 179 Вт

Это соответствует закону сохранения энергии.

Общее обсуждение

Обратите внимание, что как токи, так и мощность при параллельном подключении больше, чем для тех же устройств, подключенных последовательно.

Основные характеристики параллельных резисторов
  1. Параллельное сопротивление определяется из [latex] \ frac {1} {{R} _ {\ text {p}}} = \ frac {1} {{R} _ {1}} + \ frac {1} { {R} _ {2}} + \ frac {1} {{R} _ {3}} + \ text {…} \\ [/ latex], и оно меньше любого отдельного сопротивления в комбинации.
  2. Каждый резистор, включенный параллельно, имеет такое же полное напряжение, как и источник. (В системах распределения электроэнергии чаще всего используются параллельные соединения для питания бесчисленных устройств, обслуживаемых одним и тем же напряжением, и для того, чтобы они могли работать независимо.)
  3. Параллельные резисторы не получают суммарный ток каждый; они делят это.

Сочетания последовательного и параллельного

Более сложные соединения резисторов иногда представляют собой просто комбинации последовательного и параллельного. Они часто встречаются, особенно если учитывать сопротивление провода. В этом случае сопротивление провода включено последовательно с другими сопротивлениями, включенными параллельно. Комбинации последовательного и параллельного подключения можно свести к одному эквивалентному сопротивлению, используя методику, показанную на рисунке 4.Различные части идентифицируются как последовательные или параллельные, уменьшаются до их эквивалентов и далее уменьшаются до тех пор, пока не останется единственное сопротивление. Процесс более трудоемкий, чем трудный.

Рис. 4. Эта комбинация из семи резисторов имеет как последовательные, так и параллельные части. Каждое из них идентифицируется и приводится к эквивалентному сопротивлению, а затем уменьшается до тех пор, пока не будет достигнуто единичное эквивалентное сопротивление.

Самая простая комбинация последовательного и параллельного сопротивления, показанная на рисунке 4, также является наиболее поучительной, поскольку она используется во многих приложениях.Например, R 1 может быть сопротивлением проводов от автомобильного аккумулятора к его электрическим устройствам, которые подключены параллельно. R 2 и R 3 могли быть стартером и светом салона. Ранее мы предполагали, что сопротивление провода незначительно, но, когда это не так, оно имеет важные последствия, как показывает следующий пример.

Пример 3. Расчет сопротивления,

IR Падение, ток и рассеиваемая мощность: объединение последовательных и параллельных цепей

На рис. 5 показаны резисторы из двух предыдущих примеров, подключенные другим способом — комбинацией последовательного и параллельного подключения.Мы можем считать R 1 сопротивлением проводов, ведущих к R 2 и R 3 . (а) Найдите полное сопротивление. (b) Что такое падение IR в R 1 ? (c) Найдите текущие значения от I 2 до R 2 . (d) Какую мощность рассеивает R 2 ?

Рис. 5. Эти три резистора подключены к источнику напряжения, так что R 2 и R 3 параллельны друг другу, и эта комбинация последовательно с R 1 .

Стратегия и решение для (а)

Чтобы найти полное сопротивление, отметим, что R 2 и R 3 находятся параллельно, и их комбинация R p находится последовательно с R 1 . Таким образом, полное (эквивалентное) сопротивление этой комбинации составляет

.

R общ = R 1 + R p .

Сначала находим R p , используя уравнение для параллельных резисторов и вводя известные значения:

[латекс] \ frac {1} {{R} _ {\ text {p}}} = \ frac {1} {{R} _ {2}} + \ frac {1} {{R} _ {3 }} = \ frac {1} {6 \ text {.} \ text {00} \ text {} \ Omega} + \ frac {1} {\ text {13} \ text {.} 0 \ text {} \ Omega} = \ frac {0.2436} {\ text {} \ Омега} \\ [/ латекс].

Инвертирование дает

[латекс] {R} _ {\ text {p}} = \ frac {1} {0,2436} \ text {} \ Omega = 4.11 \ text {} \ Omega \\ [/ latex].

Таким образом, общее сопротивление равно

.

R до = R 1 + R p = 1,00 Ом + 4,11 Ом = 5,11 Ом.

Обсуждение для (а)

Общее сопротивление этой комбинации является промежуточным между значениями чистой серии и чистой параллели (20.0 Ом и 0,804 Ом соответственно), найденные для тех же резисторов в двух предыдущих примерах.

Стратегия и решение для (b)

Чтобы найти падение IR в R 1 , отметим, что полный ток I протекает через R 1 . Таким образом, падение IR составляет

.

В 1 = ИК 1

Мы должны найти I , прежде чем сможем вычислить V 1 .Полный ток I находится с помощью закона Ома для цепи. То есть

[латекс] I = \ frac {V} {{R} _ {\ text {tot}}} = \ frac {\ text {12.0} \ text {V}} {5.11 \ text {} \ Omega} = 2.35 \ text {A} \\ [/ latex].

Вводя это в выражение выше, мы получаем

В 1 = IR 1 = (2,35 А) (1,00 Ом) = 2,35 В.

Обсуждение для (б)

Напряжение, приложенное к R 2 и R 3 , меньше общего напряжения на величину В 1 .Когда сопротивление провода велико, это может существенно повлиять на работу устройств, представленных R 2 и R 3 .

Стратегия и решение для (c)

Чтобы найти ток через R 2 , мы должны сначала найти приложенное к нему напряжение. Мы называем это напряжение В p , потому что оно приложено к параллельной комбинации резисторов. Напряжение, приложенное как к R 2 , так и к R 3 , уменьшается на величину В 1 , поэтому оно равно

.

V p = V V 1 = 12.0 В — 2,35 В = 9,65 В.

Теперь ток I 2 через сопротивление R 2 находится по закону Ома:

[латекс] {I} _ {2} = \ frac {{V} _ {\ text {p}}} {{R} _ {2}} = \ frac {9.65 \ text {V}} {6.00 \ текст {} \ Omega} = 1,61 \ text {A} \\ [/ latex].

Обсуждение для (c)

Ток меньше, чем 2,00 А, которые протекали через R 2 , когда он был подключен параллельно к батарее в предыдущем примере параллельной цепи.

Стратегия и решение для (d)

Мощность, рассеиваемая на R 2 определяется на

P 2 = ( I 2 ) 2 R 2 = (1,61 A) 2 (6,00 Ом) = 15,5 Вт

Обсуждение для (d)

Мощность меньше 24,0 Вт, рассеиваемых этим резистором при параллельном подключении к источнику 12,0 В.

Одним из следствий этого последнего примера является то, что сопротивление в проводах снижает ток и мощность, подаваемую на резистор.Если сопротивление провода относительно велико, как в изношенном (или очень длинном) удлинителе, то эти потери могут быть значительными. Если потребляется большой ток, падение IR в проводах также может быть значительным.

Например, когда вы роетесь в холодильнике и включается мотор, свет холодильника на мгновение гаснет. Точно так же вы можете увидеть тусклый свет в салоне, когда вы запускаете двигатель вашего автомобиля (хотя это может быть связано с сопротивлением внутри самой батареи).

То, что происходит в этих сильноточных ситуациях, показано на рисунке 6. Устройство, обозначенное как R 3 , имеет очень низкое сопротивление, поэтому при его включении протекает большой ток. Этот повышенный ток вызывает большее падение IR в проводах, представленных R 1 , уменьшая напряжение на лампе (которое составляет R 2 ), которое затем заметно гаснет.

Рис. 6. Почему гаснет свет при включении большого прибора? Ответ заключается в том, что большой ток, потребляемый двигателем прибора, вызывает значительное падение напряжения в проводах и снижает напряжение на свету.

Проверьте свое понимание

Можно ли любую произвольную комбинацию резисторов разбить на последовательную и параллельную? Посмотрите, сможете ли вы нарисовать принципиальную схему резисторов, которые нельзя разбить на комбинации последовательно и параллельно.

Решение Нет, есть много способов подключения резисторов, которые не являются комбинациями последовательного и параллельного, включая петли и переходы. В таких случаях правила Кирхгофа, которые будут включены в Правила Кирхгофа, позволят вам проанализировать схему.

Стратегии решения проблем для последовательных и параллельных резисторов
  1. Нарисуйте четкую принципиальную схему, обозначив все резисторы и источники напряжения. Этот шаг включает список известных проблем, поскольку они отмечены на вашей принципиальной схеме.
  2. Определите, что именно необходимо определить в проблеме (определите неизвестные). Письменный список полезен.
  3. Определите, включены ли резисторы последовательно, параллельно или в комбинации последовательно и параллельно.Изучите принципиальную схему, чтобы сделать эту оценку. Резисторы включены последовательно, если через них должен последовательно проходить один и тот же ток.
  4. Используйте соответствующий список основных функций для последовательных или параллельных подключений, чтобы найти неизвестные. Есть один список для серий, а другой — для параллелей. Если ваша проблема представляет собой комбинацию последовательного и параллельного соединения, уменьшайте ее поэтапно, рассматривая отдельные группы последовательных или параллельных соединений, как это сделано в этом модуле и примерах. Особое примечание: При обнаружении R необходимо проявлять осторожность.
  5. Проверьте, являются ли ответы разумными и последовательными. Единицы и числовые результаты должны быть разумными. Общее последовательное сопротивление должно быть больше, а общее параллельное сопротивление, например, должно быть меньше. Мощность должна быть больше для тех же устройств, подключенных параллельно, по сравнению с последовательными и так далее.

Сводка раздела

Концептуальные вопросы

1. Переключатель имеет переменное сопротивление, которое почти равно нулю в замкнутом состоянии и очень велико в разомкнутом состоянии, и он включен последовательно с устройством, которым он управляет.Объясните влияние переключателя на рис. 7 на ток в разомкнутом и замкнутом состоянии.

Рис. 7. Переключатель обычно включается последовательно с источником сопротивления и напряжения. В идеале переключатель имеет почти нулевое сопротивление в замкнутом состоянии, но имеет чрезвычайно большое сопротивление в разомкнутом состоянии. (Обратите внимание, что на этой диаграмме скрипт E представляет напряжение (или электродвижущую силу) батареи.)

2. Какое напряжение на разомкнутом переключателе на Рисунке 7?

3. На разомкнутом переключателе есть напряжение, как на Рисунке 7.Почему же тогда мощность, рассеиваемая разомкнутым переключателем, мала?

4. Почему мощность, рассеиваемая замкнутым переключателем, как на Рисунке 7, мала?

5. Студент в физической лаборатории по ошибке подключил электрическую лампочку, батарею и выключатель, как показано на рисунке 8. Объясните, почему лампочка горит, когда выключатель разомкнут, и гаснет, когда выключатель замкнут. (Не пытайтесь — батарея сильно разряжается!)

Рис. 8. Ошибка подключения. Включите этот переключатель параллельно устройству, обозначенному [латекс] R [/ латекс].(Обратите внимание, что на этой диаграмме скрипт E представляет напряжение (или электродвижущую силу) батареи.)

6. Зная, что сила удара зависит от величины тока, протекающего через ваше тело, вы бы предпочли, чтобы он был включен последовательно или параллельно с сопротивлением, таким как нагревательный элемент тостера, если он потрясен им? Объяснять.

7. Были бы ваши фары тусклыми при запуске двигателя автомобиля, если бы провода в вашем автомобиле были сверхпроводниками? (Не пренебрегайте внутренним сопротивлением батареи.) Объяснять.

8. Некоторые гирлянды праздничных огней соединены последовательно для экономии затрат на проводку. В старой версии использовались лампочки, которые при перегорании прерывали электрическое соединение, как открытый выключатель. Если одна такая лампочка перегорит, что случится с остальными? Если такая цепочка работает от 120 В и имеет 40 одинаковых лампочек, каково нормальное рабочее напряжение каждой? В более новых версиях используются лампы, которые при перегорании замыкаются накоротко, как замкнутый выключатель. Если одна такая лампочка перегорит, что случится с остальными? Если такая цепочка работает от 120 В и в ней осталось 39 идентичных лампочек, каково тогда рабочее напряжение каждой?

9.Если две бытовые лампочки мощностью 60 и 100 Вт подключить последовательно к бытовой электросети, какая из них будет ярче? Объяснять.

10. Предположим, вы проводите физическую лабораторию, в которой вас просят вставить резистор в цепь, но все прилагаемые резисторы имеют большее сопротивление, чем запрошенное значение. Как бы вы соединили доступные сопротивления, чтобы попытаться получить меньшее запрошенное значение?

11. Перед Второй мировой войной некоторые радиостанции получали питание через «шнур сопротивления», который имел значительное сопротивление.Такой резистивный шнур снижает напряжение до желаемого уровня для ламп радиоприемника и т.п., и это экономит расходы на трансформатор. Объясните, почему шнуры сопротивления нагреваются и тратят энергию при включенном радио.

12. У некоторых лампочек есть три уровня мощности (не включая ноль), получаемые от нескольких нитей накала, которые индивидуально переключаются и соединяются параллельно. Какое минимальное количество нитей необходимо для трех режимов мощности?

Задачи и упражнения

Примечание. Можно считать, что данные, взятые из цифр, имеют точность до трех значащих цифр.

1. (a) Каково сопротивление десяти последовательно соединенных резисторов сопротивлением 275 Ом? (б) Параллельно?

2. (a) Каково сопротивление последовательно соединенных резисторов 1,00 × 10 2 Ом, 2,50 кОм и 4,00 кОм? (б) Параллельно?

3. Какое наибольшее и наименьшее сопротивление можно получить, соединив резисторы на 36,0 Ом, 50,0 Ом и 700 Ом?

4. Тостер на 1800 Вт, электрическая сковорода на 1400 Вт и лампа на 75 Вт подключены к одной розетке в цепи 15 А, 120 В.(Три устройства работают параллельно, если подключены к одной розетке.) а) Какой ток потребляет каждое устройство? (b) Перегорит ли эта комбинация предохранитель на 15 А?

5. Фара мощностью 30,0 Вт и стартер мощностью 2,40 кВт обычно подключаются параллельно в систему на 12,0 В. Какую мощность потребляли бы одна фара и стартер при последовательном подключении к батарее 12,0 В? (Не обращайте внимания на любое другое сопротивление в цепи и любое изменение сопротивления в двух устройствах.)

6.(a) Учитывая батарею 48,0 В и резисторы 24,0 Ом и 96,0 Ом, найдите для каждого из них ток и мощность при последовательном соединении. (b) Повторите, когда сопротивления включены параллельно.

7. Ссылаясь на пример комбинирования последовательных и параллельных цепей и рисунок 5, вычислите I 3 двумя следующими способами: (a) по известным значениям I и I 2 ; (б) используя закон Ома для R 3 . В обеих частях явно показано, как вы следуете шагам, описанным в описании стратегии решения проблем для последовательных и параллельных резисторов выше.

Рис. 5. Эти три резистора подключены к источнику напряжения, так что R 2 и R 3 параллельны друг другу, и эта комбинация последовательно с R 1 .

8. Ссылаясь на рисунок 5: (a) Вычислите P 3 и обратите внимание на его сравнение с P 3 , найденным в первых двух примерах задач в этом модуле. (b) Найдите полную мощность, отдаваемую источником, и сравните ее с суммой мощностей, рассеиваемых резисторами.

9. См. Рисунок 6 и обсуждение затемнения света при включении тяжелого прибора. (a) Учитывая, что источник напряжения составляет 120 В, сопротивление провода составляет 0,400 Ом, а номинальная мощность лампы составляет 75,0 Вт, какая мощность будет рассеиваться лампой, если при включении двигателя через провода пройдет в общей сложности 15,0 А? Предположите незначительное изменение сопротивления лампы. б) Какая мощность потребляет двигатель?

Рис. 6. Почему гаснет свет при включении большого прибора? Ответ заключается в том, что большой ток, потребляемый двигателем прибора, вызывает значительное падение напряжения в проводах и снижает напряжение на свету.

10. Линия электропередачи на 240 кВ, имеющая 5,00 × 10 2 , подвешена к заземленным металлическим опорам с помощью керамических изоляторов, каждый из которых имеет сопротивление 1,00 × 10 9 Ом (рис. 9 (а)). Какое сопротивление на землю у 100 изоляторов? (b) Рассчитайте мощность, рассеиваемую 100 из них. (c) Какая доля мощности, переносимой линией, составляет это? Ясно покажите, как вы выполняете шаги, описанные в описании стратегии решения проблем для последовательных и параллельных резисторов выше.

Рис. 9. Высоковольтная (240 кВ) линия электропередачи 5,00 × 10 2 подвешена к заземленной металлической опоре электропередачи. Ряд керамических изоляторов обеспечивает сопротивление 1,00 × 10 9 Ом каждый.

11. Покажите, что если два резистора R 1 и R 2 объединены, и один из них намного больше другого ( R 1 >> R 2 ): (a ) Их последовательное сопротивление почти равно большему сопротивлению R 1 .(б) Их параллельное сопротивление почти равно меньшему сопротивлению R 2 .

12. Необоснованные результаты Два резистора, один из которых имеет сопротивление 145 Ом, подключены параллельно, чтобы получить общее сопротивление 150 Ом. а) Каково значение второго сопротивления? б) Что неразумного в этом результате? (c) Какие предположения необоснованны или непоследовательны?

13. Необоснованные результаты Два резистора, один из которых имеет сопротивление 900 кОм, соединены последовательно, чтобы получить общее сопротивление 0.500 МОм. а) Каково значение второго сопротивления? б) Что неразумного в этом результате? (c) Какие предположения необоснованны или непоследовательны?

Глоссарий

серия:
последовательность резисторов или других компонентов, включенных в цепь один за другим
резистор:
компонент, обеспечивающий сопротивление току, протекающему через электрическую цепь
сопротивление:
вызывает потерю электроэнергии в цепи
Закон Ома:
соотношение между током, напряжением и сопротивлением в электрической цепи: В = IR
напряжение:
электрическая потенциальная энергия на единицу заряда; электрическое давление, создаваемое источником питания, например аккумулятором
падение напряжения:
потеря электроэнергии при прохождении тока через резистор, провод или другой компонент
ток:
поток заряда через электрическую цепь мимо заданной точки измерения
Закон Джоуля:
соотношение между потенциальной электрической мощностью, напряжением и сопротивлением в электрической цепи, определяемое следующим образом: [latex] {P} _ {e} = \ text {IV} [/ latex]
параллельно:
подключение резисторов или других компонентов в электрической цепи, так что каждый компонент получает одинаковое напряжение от источника питания; часто изображается на диаграмме в виде лестницы, где каждый компонент находится на ступеньке лестницы

Избранные решения проблем и упражнения

1.(а) 2,75 кОм (б) 27,5 Ом

3. (а) 786 Ом (б) 20,3 Ом

5. 29,6 Вт

7. (а) 0,74 А (б) 0,742 А

9. (а) 60,8 Вт (б) 3,18 кВт

11. (a) [латекс] \ begin {array} {} {R} _ {\ text {s}} = {R} _ {1} + {R} _ {2} \\ \ Rightarrow {R} _ {\ text {s}} \ приблизительно {R} _ {1} \ left ({R} _ {1} \ text {>>} {R} _ {2} \ right) \ end {array} \\ [/ латекс]

(b) [латекс] \ frac {1} {{R} _ {p}} = \ frac {1} {{R} _ {1}} + \ frac {1} {{R} _ {2} } = \ frac {{R} _ {1} + {R} _ {2}} {{R} _ {1} {R} _ {2}} \\ [/ latex],

, так что

[латекс] \ begin {array} {} {R} _ {p} = \ frac {{R} _ {1} {R} _ {2}} {{R} _ {1} + {R} _ {2}} \ приблизительно \ frac {{R} _ {1} {R} _ {2}} {{R} _ {1}} = {R} _ {2} \ left ({R} _ {1 } \ text {>>} {R} _ {2} \ right) \ text {.} \ end {array} \\ [/ latex]

13. (a) –400 кОм (b) Сопротивление не может быть отрицательным. (c) Считается, что последовательное сопротивление меньше, чем у одного из резисторов, но должно быть больше, чем у любого из резисторов.

Как рассчитать последовательные и параллельные резисторы — Kitronik Ltd

Резисторы серии

Когда резисторы подключаются друг за другом, это называется последовательным соединением. Это показано ниже. Чтобы рассчитать общее общее сопротивление ряда резисторов, подключенных таким образом, вы складываете отдельные сопротивления.Это делается по следующей формуле: Rtotal = R1 + R2 + R3 и так далее. Пример: чтобы рассчитать общее сопротивление для этих трех последовательно соединенных резисторов.
Rtotal = R1 + R2 + R3 = 100 + 82 + 1 Ом = 183 Ом

Задача 1:

Рассчитайте общее сопротивление следующего последовательно включенного резистора.
R Итого = _______________
= _______________
R Итого = _______________
= _______________
R Итого = _______________
= _______________

Параллельные резисторы

Когда резисторы подключаются друг к другу (бок о бок), это называется параллельным подключением.Это показано ниже.

Два параллельных резистора

Чтобы рассчитать общее полное сопротивление a двух резисторов, подключенных таким образом, вы можете использовать следующую формулу:
Пример: чтобы рассчитать полное сопротивление для этих двух резисторов, включенных параллельно.

Задача 2:

Рассчитайте полное сопротивление следующего резистора, включенного параллельно.

Три или более резистора, включенных параллельно

Для расчета общего общего сопротивления ряда из трех или более резисторов, подключенных таким образом, вы можете использовать следующую формулу: Пример: Чтобы вычислить общее сопротивление для этих трех резисторов, подключенных параллельно

Задача 3:

Рассчитайте полное сопротивление следующего резистора, включенного параллельно.

ответов

Задача 1

1 = 1492 Ом 2 = 2242 Ом 3 = 4847 Ом

Задача 2

1 = 5 Ом 2 = 9,57 Ом 3 = 248,12 Ом

Задача 3

1 = 5,95 Ом 2 = 23,76 Ом Загрузите pdf-версию этой страницы здесь. Подробнее об авторе подробнее »

© Kitronik Ltd — Вы можете распечатать эту страницу и ссылку на нее, но не должны копировать страницу или ее часть без предварительного письменного согласия Kitronik.

Калькулятор параллельных резисторов

R1 + R2 = эквивалентный резистор R схема сопротивления, эквивалентная общая сумма резисторов, упрощенная комбинация = параллельная

параллельная калькуляция резисторов R1 + R2 = эквивалентный резистор R, эквивалентная схема сопротивления, эквивалентная общая схема поиска резисторов, упрощенная комбинация = параллельная — sengpielaudio Sengpiel Berlin


R всего Формула:
R всего = R1 × R2 / (R1 + R2)

Введите , два значения резистора , будет рассчитано третье значение параллельной цепи.
Вы даже можете ввести общее сопротивление R общее и одно известное сопротивление R 1 или R 2 .

Формула (уравнение) для расчета двух сопротивлений R 1 и R 2 , соединенных параллельно:

Расчет необходимого параллельного резистора R 2 , когда задается R 1 и общее сопротивление R итого :

Решение формулы R итого = ( R 1 × R 2 ) / ( R 1 + R 2 ) для R 1 :
Первый шаг — очистить все дроби путем умножения на наименьшее
. общий знаменатель, то есть R t × R 1 × R 2 … итого получаем:
1/ R всего = 1/ R 1 + 1/ R 2
R всего × R 1 × R 2 [1/ R всего = 1/ R 1 + 1/ R 2 ]
R 1 × R 2 = R всего × R 2 + R всего × R 1 затем соберите члены с помощью R 1 и решите
R 1 × R 2 R всего × R 1 = R всего × R 2
R 1 ( R 2 R всего ) = R 2 × R всего 9001 2
Последний шаг:
R 1 = R 2 × R итого / ( 9007 9123 900 — 900 — R всего )
или:
R 2 = R 1 × 09 9128 900 всего 9128 900 R 1 R всего )

Примечание: Этот калькулятор также может решать другие математические задачи.Расчет резисторов параллельно
точно так же, как вычисления, необходимые для параллельных катушек индуктивности или для конденсаторов, включенных последовательно.

Два резистора, включенных параллельно, и результирующее общее сопротивление: Два одинаковых значения,
также покажите уравнение, что результаты всегда равны половине. Это упрощает работу, когда
проектирование схем или прототипирование. С кепками всегда вдвое больше, потом с кепками всего
просто сложите параллельно.

• Поисковые сопротивления R 1 и R 2 , когда заданное сопротивление (эквивалентное сопротивление) известно •

Расчет: пары резисторов — обратный калькулятор
Поиск R 1 и R 2 с известным целевым сопротивлением

● Рассчитать несколько резисторов параллельно ●

Этот калькулятор определяет сопротивление от до 10 резисторов, включенных параллельно .
Введите значения сопротивления в поля ниже и после ввода всех значений
. нажмите кнопку «рассчитать», и результат появится в поле под этой кнопкой.
В качестве теста, если мы введем сопротивления 4, 6 и 12 Ом, ответ должен быть 2 Ом.
Примечание. При снятии флажков вручную сохраненные значения не сбрасываются. Воспользуйтесь «сбросом».

Закон Ома — калькулятор и формулы

Два резистора, включенных параллельно, и результирующее общее сопротивление
Сопротивление в диапазоне от 1 Ом до 100 Ом

R2 R1
1 1.5 2,2 3,3 4,7 6,8 10 15 22 33 47 68
1 0,5 0,6 0,69 0.77 0,83 0,87 0,91 0,93 0,95 0,97 0,98 0,99
1,5 0,6 0,75 0,89 1,03 1,14 1,22 1,30 1,36 1,40 1,43 1.45 1,46
2,2 0,69 0,89 1,1 1,32 1,50 1,66 1,82 1,92 2,0 2,06 2,10 2,13
3,3 0,77 1,03 1,32 1.65 1,94 2,22 2,48 2,70 2,87 3,00 3,08 3,14
4,7 0,83 1,14 1,50 1,94 2,35 2,78 3,20 3,58 3,87 4,12 4.27 4,39
6,8 0,87 1,22 1,66 2,22 2,78 3,40 4,05 4,68 5,19 5,64 5,94 6,18
10 0,91 1,30 1,82 2.48 3,20 4,05 5,0 6,0 6,9 7,7 8,3 8,7
15 0,93 1,36 1,92 2,70 3,58 4,68 6,0 7,50 8,9 10,3 11,4 12.2
22 0,95 1,40 2,00 2,87 3,87 5,19 6,9 8,9 11,0 13,2 15,0 16,6
33 0,97 1,43 2,06 3,0 4.12 5,64 7,7 10,3 13,2 16,5 19,4 22,2
47 0,98 1,45 2,1 3,08 4,27 5,94 8,3 11,4 15,0 19,4 23,5 27.8
68 0,99 1,46 2,13 3,14 4,39 6,18 8,7 12,2 16,6 22,2 27,8 34,0

Примечание: Этот калькулятор также может решать другие математические задачи. Расчет резисторов параллельно
точно так же, как вычисления, необходимые для параллельных катушек индуктивности или для конденсаторов, включенных последовательно.

Мощность, рассеиваемая на резисторе: P = В × I , P = В 2 / R , P = I 2 × 9022.

Примечание: Для последовательно соединенных резисторов ток одинаков для каждого резистора,
а для резисторов, включенных параллельно, напряжение одинаково для каждого резистора.

Учебное пособие по физике: параллельные схемы

Как упоминалось в предыдущем разделе Урока 4, два или более электрических устройства в цепи могут быть соединены последовательным или параллельным соединением. Когда все устройства соединены с использованием параллельных соединений, схема называется параллельной схемой . В параллельной схеме каждое устройство помещается в свою отдельную ветвь . Наличие ответвлений означает, что существует несколько путей, по которым заряд может проходить через внешнюю цепь.Каждый заряд, проходящий через контур внешней цепи, будет проходить через единственный резистор, присутствующий в одной ветви. Прибыв в место разветвления или узел, заряд делает выбор , через какую ветвь пройти на обратном пути к терминалу с низким потенциалом.

Краткое сравнение и контраст между последовательными и параллельными цепями было сделано в предыдущем разделе Урока 4. В этом разделе было подчеркнуто, что добавление большего количества резисторов в параллельную цепь приводит к довольно неожиданному результату — уменьшению общего сопротивления. .Поскольку существует несколько путей, по которым может протекать заряд, добавление еще одного резистора в отдельную ветвь обеспечивает еще один путь, по которому заряд может проходить через основную область сопротивления в цепи. Это уменьшенное сопротивление в результате увеличения количества ветвей будет иметь эффект увеличения скорости, с которой течет заряд (также известной как ток). Чтобы сделать этот довольно неожиданный результат более разумным, была введена аналогия с платными дорогами. Плата за проезд — это основное место сопротивления автомобильному потоку на платной дороге.Добавление дополнительных пунктов сбора платы за проезд в пределах их собственного отделения на платной дороге обеспечит больше путей для автомобилей, проезжающих через станцию ​​сбора платы за проезд. Эти дополнительные пункты пропуска снизят общее сопротивление потоку автомобилей и увеличат скорость их движения.

Текущий

Скорость, с которой заряд проходит через цепь, называется током. Заряд НЕ накапливается и не начинает накапливаться в любом заданном месте, так что ток в одном месте больше, чем в других местах.Заряд НЕ расходуется резисторами таким образом, что в одном месте ток меньше, чем в другом. В параллельной схеме заряд делит на отдельные ветви, так что в одной ветви может быть больше тока, чем в другой. Тем не менее, если брать в целом, общая сумма тока во всех ветвях при сложении равна величине тока в местах за пределами ветвей. Правило, что ток везде одинаковый все еще работает, только с закруткой.Сила тока вне ветвей равна сумме токов в отдельных ветвях. Это все еще та же величина тока, только разделенная на несколько путей.

В форме уравнения этот принцип можно записать как

I всего = I 1 + I 2 + I 3 + …

, где I total — общая величина тока вне ветвей (и в батарее), а I 1 , I 2 и I 3 представляют ток в отдельных ветвях цепи.

В этом блоке широко использовалась аналогия между расходом заряда и расходом воды. Еще раз вернемся к аналогии, чтобы проиллюстрировать, как сумма текущих значений в ветвях равна сумме вне ветвей. Поток заряда в проводах аналогичен потоку воды в трубах. Рассмотрим приведенные ниже схемы, на которых поток воды в трубах делится на отдельные ответвления. В каждом узле (место разветвления) вода проходит двумя или более отдельными путями.Скорость, с которой вода поступает в узел (измеряется в галлонах в минуту), будет равна сумме расходов в отдельных ветвях за пределами узла. Точно так же, когда две или более ветви подаются в узел, скорость, с которой вода вытекает из узла, будет равна сумме расходов в отдельных ветвях, которые подаются в узел.

Тот же принцип разделения потока применяется к электрическим цепям. Скорость, с которой заряд поступает в узел, равна сумме расходов в отдельных ветвях за пределами узла.Это проиллюстрировано в приведенных ниже примерах. В примерах вводится новый символ схемы — буква A, заключенная в круг. Это символ амперметра — устройства, используемого для измерения силы тока в определенной точке. Амперметр способен измерять ток, оказывая при этом незначительное сопротивление потоку заряда.

Диаграмма A показывает два резистора, подключенных параллельно с узлами в точках A и B. Заряд течет в точку A со скоростью 6 ампер и делится на два пути — один через резистор 1, а другой — через резистор 2.Ток в ветви с резистором 1 составляет 2 ампера, а ток в ветви с резистором 2 — 4 ампера. После того, как эти две ветви снова встретятся в точке B, чтобы сформировать единую линию, ток снова станет 6 ампер. Таким образом, мы видим, что принцип, согласно которому ток вне ветвей равен сумме тока в отдельных ветвях, верен.

I всего = I 1 + I 2

6 ампер = 2 ампера + 4 ампера

Схема B выше может быть немного сложнее, если три резистора расположены параллельно.На схеме обозначены четыре узла, обозначенные буквами A, B, C и D. Заряд течет в точку A со скоростью 12 ампер и делится на два пути: один проходит через резистор 1, а другой направляется к точке B (и резисторам 2). и 3). 12 ампер тока делятся на 2-амперную (через резистор 1) и 10-амперную (в направлении точки B). В точке B происходит дальнейшее разделение потока на два пути — один через резистор 2, а другой через резистор 3. Ток в 10 ампер, приближающийся к точке B, делится на 6-амперный канал (через резистор 2) и 4-канальный. -амперный тракт (через резистор 3).Таким образом, видно, что значения тока в трех ветвях составляют 2 ампера, 6 ампер и 4 ампера, и что сумма значений тока в отдельных ветвях равна току вне ветвей.

I всего = I 1 + I 2 + I 3

12 ампер = 2 ампер + 6 ампер + 4 ампер

Анализ потока в точках C и D также может быть проведен, и будет замечено, что сумма расходов потока в этих точках равна скорости потока, находящейся непосредственно за этими точками.

Эквивалентное сопротивление

Фактическая величина тока всегда изменяется обратно пропорционально величине общего сопротивления. Существует четкая взаимосвязь между сопротивлением отдельных резисторов и общим сопротивлением набора резисторов. Чтобы исследовать эту взаимосвязь, давайте начнем с простейшего случая, когда два резистора помещены в параллельные ветви, каждый из которых имеет одинаковое значение сопротивления 4 Ом.Поскольку схема предлагает два равных путей для потока заряда, только половина заряда выберет для прохождения через данную ветвь. В то время как каждая отдельная ветвь предлагает сопротивление 4 Ом любому заряду, который проходит через нее, только половина всего заряда, протекающего по цепи, будет встречать сопротивление 4 Ом этой отдельной ветви. Таким образом, что касается батареи, которая накачивает заряд, наличие двух параллельно подключенных резисторов 4 Ом было бы эквивалентно наличию одного резистора 2 Ом в цепи.Таким же образом, наличие двух параллельно подключенных резисторов сопротивлением 6 Ом было бы эквивалентно наличию в цепи одного резистора сопротивлением 3 Ом. А наличие двух параллельных резисторов 12 Ом было бы эквивалентно наличию в цепи одного резистора 6 Ом.

Теперь давайте рассмотрим другой простой случай, когда три резистора включены параллельно, каждый из которых имеет одинаковое сопротивление 6 Ом. При трех равных путях прохождения заряда через внешнюю цепь только одна треть заряда будет проходить через данную ветвь.Каждая отдельная ветвь обеспечивает сопротивление 6 Ом проходящему через нее заряду. Однако тот факт, что только одна треть заряда проходит через определенную ветвь, означает, что общее сопротивление цепи составляет 2 Ом. Что касается батареи, которая нагнетает заряд, наличие трех параллельных резисторов 6 Ом было бы эквивалентно наличию одного резистора 2 Ом в цепи. Таким же образом, наличие трех параллельно подключенных резисторов сопротивлением 9 Ом было бы эквивалентно наличию в цепи одного резистора сопротивлением 3 Ом.А наличие трех параллельных резисторов 12 Ом было бы эквивалентно наличию одного резистора 4 Ом в цепи.

Это концепция эквивалентного сопротивления. Эквивалентное сопротивление схемы — это величина сопротивления, которая потребуется одному резистору, чтобы сравняться с общим эффектом от набора резисторов, присутствующих в схеме. Для параллельных цепей математическая формула для вычисления эквивалентного сопротивления (R eq ) составляет

. 1 / R экв. = 1 / R 1 + 1 / R 2 + 1 / R 3 +…

, где R 1 , R 2 и R 3 — значения сопротивления отдельных резисторов, подключенных параллельно. Приведенные выше примеры можно рассматривать как простые случаи, в которых все пути обладают одинаковым сопротивлением отдельному заряду, который проходит через них. Приведенные выше простые случаи были выполнены без использования уравнения. Тем не менее, уравнение подходит как для простых случаев, когда резисторы ответвления имеют одинаковые значения сопротивления, так и для более сложных случаев, когда резисторы ответвления имеют разные значения сопротивления.Например, рассмотрим применение уравнения к одному простому и одному сложному случаю ниже.

Случай 1 : Три резистора 12 Ом включены параллельно

1 / R экв = 1 / R 1 + 1 / R 2 + 1 / R 3

1 / R экв = 1 / (12 Ом) + 1 / (12 Ом) + 1 / (12 Ом)

Использование калькулятора …

1 / R экв. = 0,25 Ом -1

R экв = 1 / (0,25 Ом -1 )

R экв = 4,0 Ом

Случай 2 : резисторы 5,0 Ом, 7,0 Ом и 12 Ом включены параллельно

1 / R экв = 1 / R 1 + 1 / R 2 + 1 / R 3

1 / R экв = 1 / (5.0 Ом) + 1 / (7,0 Ом) + 1 / (12 Ом)

Использование калькулятора …

1 / R экв. = 0,42619 Ом-1

R экв = 1 / (0,42619 Ом -1 )

R экв. = 2,3 Ом


Ваша очередь попробовать

Нужно больше практики? Используйте Два параллельных резистора , виджет ниже, чтобы попробовать некоторые дополнительные проблемы.Введите любые два желаемых значения сопротивления. Используйте свой калькулятор, чтобы определить значения рэндов и рандов. Затем нажмите кнопку Отправить , чтобы проверить свои ответы. Попробуйте столько раз, сколько хотите, с разными значениями сопротивления.

Падения напряжения для параллельных ответвлений

В разделе «Схемы» учебного пособия «Физический класс» подчеркивалось, что любое повышение напряжения, полученное за счет заряда в батарее, теряется из-за заряда, когда он проходит через резисторы внешней цепи.Общее падение напряжения во внешней цепи равно увеличению напряжения при прохождении заряда по внутренней цепи. В параллельной схеме заряд не проходит через каждый резистор; скорее, он проходит через единственный резистор. Таким образом, полное падение напряжения на этом резисторе должно соответствовать напряжению батареи. Не имеет значения, проходит ли заряд через резистор 1, резистор 2 или резистор 3, падение напряжения на резисторе, которое выбирает для прохождения , должно равняться напряжению батареи.В форме уравнения этот принцип может быть выражен как

В аккумулятор = В 1 = В 2 = В 3 = …

Если три резистора размещены в параллельных ветвях и питаются от 12-вольтовой батареи, то падение напряжения на каждом из трех резисторов составляет 12 вольт. Заряд, протекающий по цепи, встретит только один из этих трех резисторов и, таким образом, столкнется с одним падением напряжения на 12 вольт.

Диаграммы электрических потенциалов были представлены в Уроке 1 этого устройства и впоследствии использовались для иллюстрации последовательных падений напряжения, происходящих в последовательных цепях.Диаграмма электрического потенциала — это концептуальный инструмент для представления разности электрических потенциалов между несколькими точками электрической цепи. Рассмотрим приведенную ниже принципиальную схему и соответствующую диаграмму электрических потенциалов.

Как показано на диаграмме электрических потенциалов, все позиции A, B, C, E и G имеют высокий электрический потенциал. Одиночный заряд выбирает только один из трех возможных путей; таким образом, в позиции B один заряд будет двигаться к точкам C, E или G, а затем пройдет через резистор, находящийся в этой ветви.Заряд не теряет свой высокий потенциал до тех пор, пока он не пройдет через резистор, либо от C к D, от E к F или от G к H. После того, как он пройдет через резистор, заряд вернется почти до 0 вольт и вернется к отрицательному значению. клемму аккумуляторной батареи для повышения ее напряжения. В отличие от последовательных цепей, заряд в параллельной цепи встречает единственное падение напряжения на своем пути через внешнюю цепь.

Ток через заданную ветвь можно предсказать, используя уравнение закона Ома, падение напряжения на резисторе и сопротивление резистора.Поскольку падение напряжения на каждом резисторе одинаково, фактором, определяющим, что резистор имеет наибольший ток, является сопротивление. Резистор с наибольшим сопротивлением испытывает наименьший ток, а резистор с наименьшим сопротивлением — наибольший ток. В этом смысле можно сказать, что заряд (как и люди) выбирает путь наименьшего сопротивления. В форме уравнения это может быть указано как

I 1 = Δ V 1 / R 1 I 2 = Δ V 2 / R 2 I 3 = Δ V 3 / R 3

Этот принцип иллюстрируется схемой, показанной ниже.Произведение I • R одинаково для каждого резистора (и равно напряжению батареи). Тем не менее, ток у каждого резистора разный. Ток наибольший там, где сопротивление наименьшее, и ток наименьший, где сопротивление наибольшее.

Математический анализ параллельных цепей

Приведенные выше принципы и формулы могут использоваться для анализа параллельной цепи и определения значений тока и разности электрических потенциалов на каждом из резисторов в параллельной цепи.Их использование будет продемонстрировано математическим анализом схемы, показанной ниже. Цель состоит в том, чтобы использовать формулы для определения эквивалентного сопротивления цепи (R eq ), тока через батарею (I до ), а также падений напряжения и тока для каждого из трех резисторов.

Анализ начинается с использования значений сопротивления отдельных резисторов для определения эквивалентного сопротивления цепи.

1 / R экв = 1 / R 1 + 1 / R 2 + 1 / R 3 = (1/17 Ω) + (1/12 Ω) + (1/11 Ω)

1 / R экв = 0.23306 Ом -1

R экв = 1 / (0,23306 Ом -1 )

R экв. = 4,29 Ом

(округлено от 4,29063 Ом)

Теперь, когда известно эквивалентное сопротивление, ток в батарее можно определить с помощью уравнения закона Ома. При использовании уравнения закона Ома (ΔV = I • R) для определения тока в батарее важно использовать напряжение батареи для ΔV и эквивалентное сопротивление для R.Расчет показан здесь:

I до = ΔV аккумулятор / R eq = (60 В) / (4,29063 Ом)

I до = 14,0 А

(округлено от 13,98396 А)

Напряжение батареи 60 В представляет собой усиление электрического потенциала за счет заряда, проходящего через батарею. Заряд теряет такое же количество электрического потенциала при любом прохождении через внешнюю цепь.То есть падение напряжения на каждом из трех резисторов такое же, как и напряжение, полученное в батарее:

ΔV аккумулятор = ΔV 1 = ΔV 2 = ΔV 3 = 60 В

Осталось определить три значения — ток каждого отдельного резистора. Закон Ома снова используется для определения значений тока для каждого резистора — это просто падение напряжения на каждом резисторе (60 В), деленное на сопротивление каждого резистора (указанное в формулировке задачи).Расчеты показаны ниже.

I 1 = ΔV 1 / R 1

I 1 = (60 В) / (17 Ом)

I 1 = 3,53 А

I 2 = ΔV 2 / R 2

I 2 = (60 В) / (12 Ом)

I 2 = 5,00 А

I 3 = ΔV 3 / R 3

I 3 = (60 В) / (11 Ом)

Я 3 = 5.45 ампер

Для проверки точности выполненных математических расчетов целесообразно проверить, удовлетворяют ли вычисленные значения принципу, согласно которому сумма значений тока для каждого отдельного резистора равна общему току в цепи (или в батарее). . Другими словами, I tot = I 1 + I 2 + I 3 ?

Является ли I tot = I 1 + I 2 + I 3 ?

Из 14.0 ампер = 3,53 ампер + 5,00 ампер + 5,45 ампер?

14,0 А = 13,98 А?

Да !!

(Разница в 0,02 ампер — это просто результат предыдущего округления значения I до от 13,98.)

Математический анализ этой параллельной цепи включал смесь концепций и уравнений. Как это часто бывает в физике, отделение понятий от уравнений при принятии решения физической проблемы является опасным актом.Здесь необходимо учитывать концепции, согласно которым падение напряжения на каждом из трех резисторов равно напряжению батареи, и что сумма тока в каждом резисторе равна общему току. Эти представления необходимы для завершения математического анализа. В следующей части Урока 4 будут исследованы комбинированные или составные схемы, в которых одни устройства включены параллельно, а другие — последовательно.

Создавайте, решайте и проверяйте свои собственные проблемы с помощью виджета Equivalent Resistance ниже.Создайте себе проблему с любым количеством резисторов и любыми номиналами. Решать проблему; затем нажмите кнопку «Отправить», чтобы проверить свой ответ.

Мы хотели бы предложить … Зачем просто читать об этом и когда можно с этим взаимодействовать? Взаимодействовать — это именно то, что вы делаете, когда используете одно из интерактивных материалов The Physics Classroom. Мы хотели бы предложить вам совместить чтение этой страницы с использованием нашего интерактивного средства построения цепей постоянного тока.Вы можете найти его в разделе Physics Interactives на нашем сайте. Построитель цепей постоянного тока предоставляет учащемуся набор для построения виртуальных цепей. Вы можете легко перетащить источники напряжения, резисторы и провода на рабочее место, а также расположить и подключить их так, как захотите. Вольтметры и амперметры позволяют измерять падение тока и напряжения. Нажатие на резистор или источник напряжения позволяет изменять сопротивление или входное напряжение. Это просто. Это весело. И это безопасно (если вы не используете его в ванне).


Проверьте свое понимание

1. По мере того, как в цепь добавляется все больше и больше резисторов, эквивалентное сопротивление цепи ____________ (увеличивается, уменьшается) и общий ток цепи ____________ (увеличивается, уменьшается).

2.Три одинаковых лампочки подключены к D-ячейке, как показано ниже. P, Q, X, Y и Z обозначают местоположения вдоль цепи. Какое из следующих утверждений верно?

а. Ток в точке Y больше, чем ток в точке Q.

г. Ток на Y больше, чем на P.

.

г. Ток в точке Y больше, чем ток в точке Z.

г. Ток в точке P больше, чем ток в точке Q.

.

e.Ток на Q больше, чем на P.

.

ф. Сила тока одинакова во всех местах.

3. Три одинаковые лампочки подключены к D-ячейке, как показано ниже. P, Q, X, Y и Z обозначают местоположения вдоль цепи. В каком месте (ах), если таковые имеются, будет ток …

а. … так же, как у X?

г…. такой же, как у Q?

г. … так же, как у Y?

г. … меньше, чем у Q?

e. … меньше, чем у P?

ф. … вдвое больше, чем у Z?

г. … в три раза больше, чем в Y?

4. Какие изменения можно внести в схему ниже, чтобы уменьшить ток в ячейке? Перечислите все подходящие варианты.

а. Увеличьте сопротивление лампы X.

г. Уменьшите сопротивление лампы X.

.

г. Увеличьте сопротивление лампы Z.

.

г. Уменьшите сопротивление лампы Z.

.

e. Увеличьте напряжение ячейки (как-нибудь).

ф. Уменьшите напряжение ячейки (как-нибудь).

г. Снять лампу Y.

.

5.Аккумулятор на 12 В, резистор на 12 Ом и резистор на 4 Ом подключаются, как показано на рисунке. Ток в резисторе 12 Ом равен ____ току в резисторе 4 Ом.

а. 1/3

г. 1/2

г. 2/3

г. то же, что

e.1,5 раза

ф. дважды

г. трижды

ч. четыре раза


6. Аккумулятор на 12 В, резистор на 12 Ом и резистор на 4 Ом подключаются, как показано.Падение напряжения на резисторе 12 Ом равно ____ падению напряжения на резисторе 4 Ом.

а. 1/3

г. 1/2

г. 2/3

г. то же, что

e. 1,5 раза

ф.дважды

г. трижды

ч. четыре раза

7. Аккумулятор на 12 В и резистор на 12 Ом подключаются, как показано на схеме. Резистор на 6 Ом добавлен к резистору на 12 Ом, чтобы создать цепь Y, как показано.Падение напряжения на резисторе 6 Ом в цепи Y равно ____ падению напряжения на резисторе X.

а. больше, чем

г. меньше

г. то же, что

8. Используйте свое понимание эквивалентного сопротивления, чтобы заполнить следующие утверждения:

а. Два резистора сопротивлением 6 Ом, помещенные параллельно, обеспечат сопротивление, эквивалентное сопротивлению одного резистора _____ Ом.

г. Три резистора 6 Ом, помещенные параллельно, обеспечат сопротивление, эквивалентное одному резистору _____ Ом.

г. Три резистора сопротивлением 8 Ом, помещенные параллельно, обеспечат сопротивление, эквивалентное сопротивлению одного резистора _____ Ом.

г. Три резистора с сопротивлением 2 Ом, 4 Ом и 6 Ом размещены параллельно. Они обеспечили бы сопротивление, эквивалентное одному резистору _____ Ом.

e. Три резистора с сопротивлением 5 Ом, 6 Ом и 7 Ом размещены параллельно.Они обеспечили бы сопротивление, эквивалентное одному резистору _____ Ом.

ф. Три резистора с сопротивлением 12 Ом, 6 Ом и 21 Ом размещены параллельно. Они обеспечили бы сопротивление, эквивалентное одному резистору _____ Ом.

9. На основании ваших ответов на вышеуказанный вопрос заполните следующую формулировку:

Общее или эквивалентное сопротивление трех параллельно включенных резисторов будет _____.

а. больше, чем сопротивление самого большого значения R.

г. меньше, чем сопротивление наименьшего значения R из трех.

г. где-то между наименьшим значением R и наибольшим значением R.

г. … ерунда! Такого обобщения сделать нельзя. Результаты меняются.

10. Три резистора включены параллельно.Если поместить в цепь с источником питания 12 В. Определите эквивалентное сопротивление, полный ток цепи, падение напряжения и ток в каждом резисторе.


Серия

и параллельные резисторы

  • Изучив этот раздел, вы сможете:
  • Рассчитайте значения общего сопротивления в сетях с последовательным сопротивлением.
  • Используйте соответствующие формулы для расчета сопротивления в цепях с параллельным сопротивлением.
  • • Вычисление суммы обратных величин.
  • • Произведение над суммой.
  • Рассчитайте значения общего сопротивления в последовательных / параллельных сетях.

Расчеты в последовательно- и параллельных резисторных цепях

Компоненты, включая резисторы в цепи, могут быть соединены вместе двумя способами:

ПОСЛЕДОВАТЕЛЬНО, так что один и тот же ток течет через все компоненты, но на каждом из них может существовать разная разность потенциалов (напряжение).

ПАРАЛЛЕЛЬНО, так что одна и та же разность потенциалов (напряжение) существует на всех компонентах, но каждый компонент может проводить разный ток.

Рис. 4.2.1 Резисторы серии

Рис. 4.2.2 Параллельные резисторы

В любом случае (для резисторов) полное сопротивление той части цепи, которая содержит резисторы, может быть рассчитано с использованием методов, описанных ниже.

Возможность рассчитать суммарное (общее) значение резисторов таким способом позволяет легко вычислить неизвестные значения сопротивления, тока и напряжения для довольно сложных схем, используя относительно простые методы.Это очень полезно при поиске неисправностей.

ПЕРЕД ДАЛЬНЕЙШЕЙ ДАЛЬНОСТЬЮ ПОПРОБУЙТЕ ИСПОЛЬЗУЙТЕ ФОРМУЛЫ ДЛЯ РАСЧЕТА ОБЩИХ ЗНАЧЕНИЙ СЕРИИ И ПАРАЛЛЕЛЬНЫХ РЕЗИСТОРОВ.

Для резисторов в серии:

Суммарное сопротивление двух или более резисторов , соединенных последовательно , определяется простым сложением индивидуальных значений резисторов, чтобы найти общую сумму (R TOT ):

Для резисторов, включенных параллельно:

Для расчета общего сопротивления цепи, в которой используются параллельные резисторы, можно использовать следующую формулу.

Обратите внимание, однако, что эта формула НЕ дает вам общего сопротивления R TOT . Это дает вам ВЗАИМОДЕЙСТВИЕ R TOT или:

Это совсем другое значение — и НЕ является полным сопротивлением. Он делится на 1, деленный на рэндов TOT . Чтобы получить правильное значение для R TOT (которое будет обратным 1 / R TOT , т. Е. TOT /1, просто нажмите соответствующую клавишу на вашем калькуляторе (отмеченную 1 / x или x-1) .

Другой способ расчета параллельных цепей.

Суммарное сопротивление двух резисторов, включенных параллельно , которое не включает обратных величин, определяется по формуле:

Эту формулу часто называют «произведение над суммой».

Он рассчитывает только ДВА резистора параллельно? Ну да, но это не большая проблема. Если имеется более двух параллельных резисторов, просто выберите два из них и определите общее сопротивление для этих двух — затем используйте это общее сопротивление, как если бы это был один резистор, и составьте еще одну пару с третьим резистором.Определите новую сумму и так далее, пока вы не включите все параллельные резисторы в этой конкретной сети.

О, еще кое-что, что нужно помнить о произведении над суммой, видите скобки вокруг суммы (нижняя часть) формулы? Это означает, что вы должны решить это ДО того, как использовать его для разделения продукта (верхняя часть) на. Если вы этого не сделаете, ваш ответ будет неправильным.

Звучит сложно? Не совсем, это просто вопрос повторения, и на практике вы не часто встречаетесь с множеством параллельных сетей с гораздо более чем двумя резисторами.Тем не менее, какую формулу вы выберете, зависит от вас, взаимная или сумма продукта.

подсказки

Использование обратного метода

Если вы используете МЕТОД ВЗАИМОДЕЙСТВИЯ для параллельных цепей, НЕ ЗАБУДЬТЕ, когда вы добавили обратные величины отдельных резисторов — вы должны снова найти обратную величину 1 / R1 + 1 / R2 + 1 / R3 = 1 / R TOT , и чтобы найти R TOT , вы должны найти обратную величину 1 / R TOT .

Упрощающие схемы

Для комбинированных последовательных и параллельных цепей сначала определите участок цепи (последовательный или параллельный).Затем перерисуйте схему, заменив участок, сопротивление которого вы нашли, одним резистором. Теперь у вас есть упрощенная схема, по которой можно найти R TOT .

Вы можете использовать формулу «произведение на сумму»:

Для цепей с более чем двумя параллельными резисторами просто определите два параллельных резистора одновременно, используя формулу произведения на сумму, а затем перерисуйте схему, заменив два резистора одним резистором, значение которого является объединенным сопротивлением двух .

Теперь вы можете использовать первое комбинированное значение в качестве единственного резистора со следующим параллельным резистором и так далее. Таким образом, можно выработать большое количество параллельных резисторов с использованием произведения на сумму.

Когда все параллельные резисторы одинакового номинала.

Если несколько одинаковых параллельных резисторов подключены, общее сопротивление будет равно номиналу резистора, умноженному на обратную величину количества резисторов.

, т. Е. Два параллельных резистора 12 кОм имеют общее сопротивление

.

12K x 1/2 = 6K

Три параллельно включенных резистора 12 кОм имеют суммарное сопротивление

.

12K x 1/3 = 4K и т. Д.

Проверяю ответ

Суммарное значение любого количества параллельных резисторов всегда будет МЕНЬШЕ, чем значение наименьшего отдельного резистора в сети. Используйте этот факт, чтобы проверить свои ответы.

Серия

и параллельная комбинация

Попробуйте несколько вычислений, основанных на последовательной и параллельной цепях резисторов. Для этого вам просто нужно использовать информацию на этой странице и на странице «Советы по расчету резисторов». Вас просят вычислить общее сопротивление для каждой цепи.Вы можете выбрать, какую формулу использовать

Вы также можете получить помощь по математике, загрузив нашу бесплатную брошюру «Советы по математике».

Прежде чем начать, подумайте об этих нескольких советах. Они упростят задачу, если вы будете внимательно им следовать.

1. Разработайте ответы с помощью карандаша и бумаги; перерисуйте схему, над которой работаете.

2. Конечно, ответ — это не просто число, это будет определенное количество Ом, не забудьте указать правильную единицу (например.г. Ω, KΩ или MΩ) или ваш ответ не имеет смысла.

3. Когда вы вводите значения в калькулятор, преобразуйте все значения KΩ или MΩ в Ом с помощью клавиши EXP. Если вы здесь ошибетесь, то получите действительно глупые ответы, в тысячи раз слишком большие или слишком маленькие.

Итак, вы прочитали эти инструкции, и вы готовы к работе. Вот способ решить типичную проблему на бумаге, чтобы (с практикой) вы не запутались.

Пример последовательной и параллельной цепи.

Хорошо, здесь есть что вспомнить, так почему бы не попробовать несколько практических вопросов в модуле резисторов 4.5 по определению общего сопротивления некоторых цепей резисторов?

резисторов, включенных последовательно и параллельно | Комбинации резисторов

Результаты обучения

  • Рассчитайте общее сопротивление различных комбинаций резисторов, т. Е. Последовательных, параллельных и последовательно-параллельных.
  • Покажите, как резисторы используются в качестве делителей напряжения и тока.
  • Рассчитайте сопротивление и значение мощности для последовательного резистора, понижающего напряжение.

Отдельные резисторы могут быть соединены вместе последовательно, параллельно или в комбинации последовательно и параллельно. Это приводит к более сложной схеме, полное сопротивление которой представляет собой комбинацию отдельных резисторов.

Комбинация резисторов серии

Для подключения резисторов серии , они соединяются встык вместе в одну линию, как показано для Рисунок 1 .Характеристики последовательно соединенных резисторов можно резюмировать следующим образом:

  • Общее сопротивление цепи ( R T ) увеличивается при последовательном подключении дополнительных резисторов и уменьшается при удалении резисторов.
  • Чтобы определить общее сопротивление цепи, просто найдите сумму отдельных сопротивлений нагрузок.
  • В этом примере, если резисторы обозначены как R 1 , R 2 и R 3 , то общее сопротивление R T рассчитывается по формуле

Рисунок 1 Резисторы, подключенные последовательно.

ПРИМЕР 1

Проблема: Три резистора, R 1 (4 Ом), R 2 (50 Ом) и R 3 (75 Ом) подключены последовательно, как показано на Рисунок 2 . Определите значение общего сопротивления комбинированной цепи.

Рисунок 2 Схема для примера 1.

Решение:

Резисторы, соединенные последовательно, используются в качестве делителей напряжения , , как показано на схеме Рисунок 3 .Делители напряжения широко используются в цепях, где один источник напряжения должен обеспечивать несколько разных значений напряжения для разных частей цепи.

Характеристики схемы последовательного делителя напряжения можно резюмировать следующим образом:

  • Через каждый резистор протекает одинаковое количество тока.
  • Входное напряжение делится пропорционально между последовательно соединенными резисторами.
  • Падение напряжения на резисторе в последовательной цепи прямо пропорционально сопротивлению резистора.
  • Чем выше значение сопротивления, тем больше падение напряжения.

Рисунок 3 Схема делителя напряжения.

Для схемы делителя напряжения падение напряжения на каждом резисторе обычно является фактором, который необходимо определить. Падение напряжения на любом резисторе пропорционально отношению его сопротивления к общему сопротивлению цепи.

Формула делителя напряжения позволяет рассчитать падение напряжения на любом из последовательно подключенных резисторов без необходимости сначала рассчитывать значение тока цепи.Заявлено в виде формулы:

ПРИМЕР 2

Резисторы R 1 (5 кОм), R 2 (3 кОм) и R 3 (2 кОм) ) соединены последовательно, образуя делитель напряжения, как показано на Рисунок 4 . Если на схему подается входное напряжение 9 вольт, рассчитайте значение падения напряжения на каждом из резисторов, используя формулу делителя напряжения.

Рисунок 4 Схема для примера 2.

Решение:

ПРИМЕР 3

У вас есть источник на 120 В, и вы хотите использовать последовательно понижающий резистор в сочетании с контрольной лампой 6 В при 150 мА для индикации подачи питания ( Рисунок 5 ). Определите значение сопротивления падения серии и требуемую мощность.

Рисунок 5 Схема для примера 3.

Решение:

Параллельная комбинация резисторов

Резисторы соединены параллельно путем соединения бок о бок как показано на Рисунок 6 .Обратите внимание, что два конца резисторов подключены к одним и тем же двум точкам.

Характеристики параллельно соединенных резисторов можно резюмировать следующим образом:

  • Общее сопротивление (R T ) сформированной цепи на меньше , чем сопротивление самого низкого значения сопротивления, присутствующего в любой из ветвей.
  • Каждый резистор обеспечивает отдельный параллельный путь для прохождения тока.
  • Если у вас есть несколько резисторов одинакового номинала, подключенных параллельно, то общее сопротивление легче всего найти, разделив общее значение сопротивления на количество подключенных резисторов.Для трех резисторов по 150 Ом, включенных параллельно, общее сопротивление составляет

Рисунок 6 Резисторы, подключенные параллельно.

Чтобы найти полное сопротивление двух неравных значений резисторов, соединенных параллельно (очень распространенное использование), используется формула произведения на сумму. Эта формула:

ПРИМЕР 4

Проблема: Резистор сопротивлением 60 Ом подключен параллельно с резистором 40 Ом, как показано на рис. 7 .Определите значение общего комбинированного сопротивления двух компонентов, используя формулу «произведение на сумму».

Рисунок 7 Схема для примера 4.

Решение:

Формула произведения на сумму лучше всего работает для двух резисторов, включенных параллельно. Если параллельно подключено более двух резисторов, использование становится труднее и менее практичным. Для более чем двух резисторов разных номиналов, соединенных параллельно, используется общая формула для полного сопротивления параллельной цепи.Эта формула:

ПРИМЕР 5

Проблема: Три резистора, R 1 (120 Ом), R 2 (60 Ом) и R 3 ( 40 Ом) подключены параллельно, как показано на Рисунок 8 . Определите значение общего сопротивления комбинированной цепи.

Рисунок 8 Схема для примера 5.

Решение:

Параллельные резистивные цепи можно рассматривать как делители тока , потому что ток разделяется или делится между различными резисторами, как показано на рисунке 9 .

Характеристики схемы параллельного делителя тока можно резюмировать следующим образом:

  • Ток, протекающий через каждый резистор ответвления, обратно пропорционален его значению сопротивления.
  • Чем меньше значение сопротивления, тем больше ток, и наоборот.
  • Резисторы с одинаковым сопротивлением пропускают через них одинаковый ток.
  • Формула, описывающая делитель тока, аналогична формуле для делителя напряжения и может быть выражена следующим образом:

Рисунок 9 Схема делителя тока.

ПРИМЕР 6

Проблема: Резисторы R 1 R 2 и R 3 (2 Ом, 3 Ом и 6 Ом соответственно) подключены параллельно, как показано на Рисунок 10 . Используйте формулу делителя тока, чтобы вычислить значение тока, протекающего через каждый из нагрузочных резисторов, если общий ток, протекающий в цепи, составляет 10 ампер.

Рисунок 10 Схема для примера 6.

Решение:

ПРИМЕР 7

Проблема: При параллельном подключении дополнительных нагрузок общее сопротивление цепи уменьшается. Для цепи, показанной на рис. 11 , определите полное сопротивление цепи при каждом из следующих рабочих условий:

  1. Переключатели 1 и 2 замкнуты.
  2. Переключатели 1, 2 и 3 замкнуты.

Рисунок 11 Схема для примера 7.

Решение:

Последовательно-параллельная комбинация резисторов

Комбинированные резистивные цепи, иначе известные как последовательно-параллельные резистивные цепи , объединяют резисторы последовательно с резисторами, включенными параллельно, как показано на рисунке 12 .

Правила, регулирующие эти цепи, те же, что и для последовательных цепей и для параллельных цепей. Сначала определяется сопротивление совокупного полного сопротивления параллельной части.Затем общее сопротивление параллельной части добавляется к любому последовательному сопротивлению, чтобы найти общее сопротивление последовательно-параллельной комбинированной цепи.

Рисунок 12 Последовательно-параллельное соединение резисторов.

ПРИМЕР 8

Проблема: Резистор 9 Ом, R 1 , и резистор 60 Ом, R 2 , подключены параллельно друг другу и последовательно с резистор 40 Ом, R 3 , как показано на рис. 13 .Определите общее сопротивление этой последовательно-параллельной комбинации резисторов.

Рисунок 13 Цепь для примера 8.

Решение:

ПРИМЕР 9

Проблема: Показания сопротивления можно использовать для проверки цепей на наличие неисправностей. Как определено в предыдущем примере, нормальное полное сопротивление этой последовательно-параллельной схемы , рис. 14, составляет 60 Ом.

  1. Найдите новое значение R T , если резистор R 1 выйдет из строя разомкнут , а значения сопротивления R 2 и R 3 останутся такой же.
  2. Аналогичным образом найдите новое значение R T , если резистор R 3 будет поврежден закорочен , тогда как значения сопротивления R 1 и R 2 остаются одинаковый.

Рисунок 14 Цепь для примера 9.

Решение:

  1. При разомкнутом отказе R 1 цепь будет состоять из 3 R последовательно с R 2 и всего сопротивление будет:

  1. При коротком замыкании R 3 цепь будет состоять из R 1 , подключенных параллельно с R 2 , а общее сопротивление будет:

Контрольные вопросы

  1. Рассчитайте общее сопротивление для каждой из следующих цепей резисторов:
    1. Последовательная цепь: R1 = 40 Ом, R2 = 75 Ом
    2. Параллельная цепь: R1 = 200 Ом, R2 = 200 Ом, R3 = 200 Ом
    3. Последовательная цепь: R1 = 2000 Ом, R2 = 6000 Ом, R3 = 2200 Ом
    4. Параллельная цепь: R1 = 14 Ом, R2 = 32 Ом
    5. Последовательная цепь: R1 = 4700 Ом, R2 = 800 Ом, R3 = 200 Ом
    6. Параллельная цепь: R1 = 60 Ом, R2 = 30 Ом, R3 = 15 Ом
  2. 9 0275 Резисторы R 1 , R 2 и R 3 (50 Ом, 30 Ом и 20 Ом соответственно) подключены последовательно через приложенное напряжение 200 В, чтобы сформировать делитель напряжения.Используя формулу делителя напряжения, рассчитайте напряжения E 1 , E 2 и E 3 .
  3. Полный ток на два параллельно соединенных резистора составляет 3 А. Сопротивление R 1 составляет 10 Ом, а сопротивление R 2 составляет 40 Ом. Используя формулу делителя тока, рассчитайте токи I 1 и I 2 .
  4. Резистор 5 Ом, R 1 , и резистор 20 Ом, R 2 , подключены параллельно друг другу и последовательно с резистором 6 Ом, R 3 .Вычислите полное сопротивление этой последовательно-параллельной цепи.
  5. Вам даны три резистора по 100 Ом, которые нужно соединить вместе. Опишите три возможные конфигурации цепей и вычислите их значения общего сопротивления.

Контрольные вопросы — ответы

  1. (a) 115 Ом, (b) 66,7 Ом, (c) 10 200 Ом, (d) 9,74 Ом, (e) 5700 Ом, (f) 8,57 Ом
  2. E 1 = 100 В, E 2 = 60 В, E 3 = 40 В
  3. I 1 = 2.

Добавить комментарий

Ваш адрес email не будет опубликован. Обязательные поля помечены *